Download as pdf or txt
Download as pdf or txt
You are on page 1of 27

1/9/14

Civil Services Main Exam 2013


Student Login / Register
Catch Us On

Home

Programs

Enroll Online

Center Locator

About CL Educate

News Notifications Analysis Events Results

Civil Services Main Exam 2013


You are here - Home - Civil Services Main 2013

Exam Analysis

Actual Papers

Solutions

Compulsory English Paper Solution 3. (a) The author feels that as more and more time-saving devices such as computers, cell phones, etc. enter our lives, more and more desperate have we become to escape them. In future, blinking machines and streaming videos will engulf our lives to such an extent that freedom will become a scarcity. Alone time will become a scarcity and in fact, more valuable, as there will be less of it available in the future. (b) Today, people have lost touch with themselves and their lives have become empty as a result of the constant inflow of data in their lives. They are slowly losing their freedom and control over their lives and barely have any time or space to think for themselves or those around them. (c) The author feels that nowadays, people are so busy in work that they hardly have any time left for their personal lives. So many time-saving devices like Internet and mobiles have entered peoples lives, leaving little or no time for recreation. People are losing the moral and emotional clarity of their thoughts. Hence, he says, We have more and more ways to communicate, but less and less to say. (d) People are taking an active interest in old-age fads such as yoga and meditation in order to get away from the world of the high-speed devices and streaming videos. These devices have eaten away into the personal space of people and have left little or no time for simple pleasures of life. People hope that spending time in a peaceful setting would help to make them calmer and sharper. They want to search that emotional and moral clarity of their lives which they have lost, being part of this modern (tech-obsessed) world. (e) The modern man has become so tech savvy and engrossed in the world of high-speed Internet that he barely has time to empathize with others. The emotion of empathy requires an individual to be calm, whereas the modern man is living in the fast lane, always in a rush to meet deadlines.

4. (a) (i) desultory lacking a plan, purpose or enthusiasm The candidates speech was marked by desultory ideas, going on to show how ill-prepared he was. (ii) grapevine literally, it refers to a climbing plant on which grapes grow. However, figuratively it refers to an informal way of spreading information or rumors through conversation. I heard about their affair through the office grapevine. (iii) holistic something that emphasizes on the importance of the whole/ complete system and the interdependence of its parts. Our leaders need to adopt a more holistic approach to achieving nation-wide growth so as to ensure that even the rural areas are uplifted. (iv) insidious causing harm in a way that is gradual or not easily noticed Rheumatoid arthritis is an insidious disease that slowly eats away your joints. (v) intransigence uncompromising or unwilling to change ones views or to agree about something Her parents gave in to her intransigence and allowed her to pursue her dreams of becoming an actor. (vi) paradigm - a typical model or pattern of something; a theory or a group of ideas about how something should be done, made, or thought about Free education for children belonging to lower income groups will bring about a paradigm change in education as well as the society. (vii) susceptible - likely or liable to be influenced or harmed by a particular thing Londoners are more susceptible to depression due to the continuous rains and cold weather. (viii) ubiquitous - present or found everywhere The companys advertisements are ubiquitous, its sales are bound to go up. (ix) voracious having a huge appetite The bloody dragon is a voracious beast that will keep seeking bloodier meat. (x) venerable - valued and respected because of old age or wisdom A venerable politician is known by his deeds and accomplishments and not his family background.

www.careerlauncher.com/news/2013/UPSCmains2013GSanalysis.html

1/27

1/9/14

Civil Services Main Exam 2013


4. (b) (i) The word one functions as a pronoun in the given sentence. In order to maintain consistency, the same pronoun should be used throughout the sentence. The correct sentence is One must do what one thinks best. (ii) The given sentence is in future tense, whereas the verb did is a past tense form of the verb do. The correct sentence is He will surely not do that, will he? (iii) When seeking permission from someone, one should use words such as may or might and not can or could. The correct sentence is May I leave the room now, sir? (iv) Phrases that start with along with are called interrupters and are not part of the main subject. In the given sentence, neighbor, which is a singular noun, is the subject. Hence, singular verb was will be used. Also, the given sentence is in past tense, hence the phrase is stalled will be replaced by was stalled. The correct sentence is My neighbor, along with two of his friends, was pushing his car, which was stalled. (v) The correct sentence is I thought of helping him, but he did not welcome my suggestion. It is grammatically incorrect to say I thought to (vi) The preposition in should be replaced by on. In is used to indicate location/ position inside/within something, for example, the book is lying in the cupboard. On is used to indicate a position that is supported by the top surface of something, for example, the book is lying on the table. Also, to maintain the correct parallel structure, the -ing forms of verbs have been used in the sentence (eating and sitting). The correct sentence is When I was a child, I enjoyed eating ice cream while sitting on the bench. (vii) Phrases that start with along with are called interrupters and are not part of the main subject. In the given sentence, Principal, which is a singular noun, is the subject. Hence, singular verb is will be used. The correct sentence is The Principal, along with the teachers, is planning to apply for a leave. (viii) When question-indicating words such as who, what, why and where appear in the middle of an interrogative sentence, the verb that follows them is usually placed at the end of the sentence. For example, Do you have an idea where she is? The correct sentence is Do you have an idea who that man is? (ix) Phrases that start with as well as are called interrupters and are not part of the main subject. In the given sentence, Unemployment, which is a singular noun, is the subject. Hence, singular verb influences will be used. The correct sentence is Unemployment as well as poverty influences the votes. (x) A Past Perfect form of the verb expresses the idea that something occurred before another action in the past. In the given sentence, the man had disappeared before I woke up. Hence, the past perfect form of the verb disappear i.e. had disappeared should be used. The correct sentence is When I woke up, the man had already disappeared after committing murder in the running train. 4. (c) (i) No matter what she says, no one dares to criticize her for it. (ii) Radha, being the eldest, had to look after her parents. (iii) She decorated the room so that it looked beautiful. (iv) The young tourist exclaimed that it was a beautiful day. (v) It was determined by the one-man committee that no action needed to be taken by them. (vi) The proud father exclaimed that his son was a wonderful batsman. (vii) My mother remarked were the children not lovely. (viii) The streets were filled with tourists during the world cup. (ix) How wonderful it would be if we go to Shimla! (x) Had I gone out earlier, I would have finished the work. 4. (d) (i) Even thoughhe was late for the meeting, his boss didnt get angry. (ii) Ashe was late for the meeting, his boss became angry. (iii) Ill not get angry, provided you are not late for the meeting. (iv) I started early but I was still late for the meeting. (v) Ill give you her telephone number in case you have problems finding her place. 4. (e) (i) It would be a piece of cake to impress Ramesh Mohan. (ii) It is a very good idea to visit old people in hospitals. (iii) It is bad for your health to smoke 20 cigarettes a day. (iv) It could be very dangerous to chat with strangers on the computer. (v) It is a waste of electricity to keep the computer on all day, says my father. 4. (f) (i) My teacher said that I need to practise more. Practise is a verb, while practice is a noun. In the above sentence, the infinitive form of verb i.e. to practise has been used. Example using practice It is a good practice to revise before examination. (ii) The hot weather affects people in different ways. Affect is a verb, while effect is a noun. In the above sentence, we use a verb to show the effect of the hot weather (on people). Example using effect The effects of women empowerment are far-reaching. (iii) Wouldnt it be nice if we had to work only on alternate days? Alternate means every other or occurring or succeeding by turns, for example, the postman comes every alternate day. On the other hand, alternative is a second option that does not replace the first, for example, CNG is an alternative for people who want to save the money spent on petrol. (iv) Our politicians are known to evade taxes. Evade means to escape or stay away from doing something, while avoid means to stay away from something. Example using avoid I want to avoid unnecessary media attention. Example using evade I evade media attention by wearing a disguise. (v) The Principal complimented the students on their fine performance. Compliment refers to an admiring remark, while complement refers to something that completes something else or makes it better, for example, the belt is a perfect complement to her outfit.

www.careerlauncher.com/news/2013/UPSCmains2013GSanalysis.html

2/27

1/9/14

Civil Services Main Exam 2013


4. (g) (i) An amateur is someone who engages in a pursuit, study, science, or sport as a pastime rather than as a profession. Some of its opposites are expert, professional, specialist, etc. (ii) Modesty is the quality of not being too proud or confident about oneself or ones abilities. Some of its opposites are arrogance, boastfulness, pompousness, conceitedness, egotism, etc. (iii) Shallow, literally means having a small distance to the bottom from the surface or highest point. However, figuratively it means not caring about or involving serious or important things. Some of its opposites are deep, profound, bottomless, superficial, etc. (iv) Conceited means having or showing too much pride in your own worth or goodness. Some of its opposites are humble, modest, egoless, etc. (v) Atheist is a person who does not believe in the existence of God. Some of its opposites are theist, believer, etc.

Solution of Civil Services (Main) Examination, 2013, PaperI Q1. Though not very useful from the point of view of a connected political history of South India, the Sangam literature portrays the social and economic conditions of its time with remarkable vividness. Comment. Ans: Sangam Literature refers to a body of classical Tamil literature created between the years c. 600 BCE to 300 CE. The period during which these poems were composed is commonly referred to as the Sangam period. The Tamil Sangams were academies, where Tamil poets and authors are said to have gathered periodically to publish their works. The literature was primarily secular dealing with everyday themes in a Tamilakam context. It deals with emotional and material topics such as love, war, governance, trade and bereavement. It narrates about classification of sectional division of the concurrent society. It informs about different religions (like Shaivism, Vaishnavism and Jainism etc.), various festivals (like worship of pattini), social traditions and customs prevalent in concurrent society, status of women, prevalent dresses and ornaments, mutual behavior of the people towards one other. It narrates about concurrent economic life on a large scale. Division of land according to its fertility and use, like mullai (forests), marutham (agricultural land), neithal (coastal regions) and paalai (deserts) is revealed by Sangam Literature. It informs about prosperous internal trade, international sea trade and also about foreign trade. It narrates about cities such as Madurai, Puhar and Uraiyur, which were centers of economic and industrial activities. A large number of archeological evidence of international trade has been recovered from Arikamedu. Kalchi was famous for pearl-culture. Puhar was a port city, whereas Uraiyur and Arikamedu were significant for cotton industry. Q2 (a) Discuss the Tandava dance as recorded in the early Indian inscriptions. Ans: Tandave is a divine dance performed by the Hindu god Shiva. It is described as a vigorous dance that is the source of the cycle of creation, preservation and dissolution. In the Hindu texts, at least seven types of Tandava, namely Ananda Tandava, Tripura Tandava, Sandhya Tandava, Samhara Tandava, Kali Tandava, Uma Tandava and Gauri Tandava, are found. Shiva as Nataraja is considered the supreme lord of dance. The Rudra Tandava depicts his violent nature, first as the creator and later as the destroyer of the universe, even of death itself, whereas the Ananda Tandava depicts Shiva enjoying. It is well narrated in the Natya Shastra authored by Bharata Muni. The Bhagavata Purana talks of Krishna dancing his Tandava on the head of the serpent Kaliya. According to Jain traditions, Indra is said to have performed the Tandava in honour of Rishabha (Jain tirthankar) on the latter's birth. Q 2 (b) Chola architecture represents a high watermark in the evolution of temple architecture. Discuss. Ans: The Cholas continued the temple building traditions of the Pallava dynasty and elevated the Dravidian temple design to greater heights. They developed a typical style of temple architecture of South India known as Dravida style, complete with Vimana or Shikhara, high walls and the gateway topped by Gopuram. Vijayalaya Choleswaram temple is an example of a surviving early Chola building. The maturity and grandeur of the Chola architecture had found expression in the two magnificent temples of Thanjavur by Rajaraja Chola and Gangaikondacholapuram by Rajendra Chola. In the temple of Thanjavur, two gopurams were built for the first time. The temple of Gangaikonda Cholapuram follows the plan of the great temple of Thanjavur but in most details, the Gangaikonda Chloapuram has characteristics of its own. It had only one enclosure wall and one gopuram. The temple architecture of Cholas is considered the parameter of Dravidian temple design. Q3. Defying the barriers of age, gender and religion, the Indian women became the torch bearer during the struggle for freedom in India. Ans: The Indian women participated in the National Movement on a large scale. They joined almost all forms of the movement. From the Swadeshi Movement up to the Quit India Movement and even after independence, they played significant role. They were involved in processions, picketing and even went to jail for their struggle for freedom. Sarojini Naidu and Nalini Sengupta were elected as the President of the Indian National Congress. Anne Besant was one of the main leaders of the Home Rule Movement. A number of women participated in the election of local bodies and provincial assemblies under the British Indian Rule and worked as ministers and secretaries. They also joined the extremist and revolutionary wings of the movement. Kalpana Datta and Pritilata Waddedar joined the Chittagong armoury raid. Caption Lakshmi Sahgal and others joined the Indian National Army. They joined communist and peasant movements along with workers movements on a large scale. They also played significant role in the movements for social and religious reforms. They struggled for the abolition of

www.careerlauncher.com/news/2013/UPSCmains2013GSanalysis.html

3/27

1/9/14

Civil Services Main Exam 2013


social evils including untouchability, reform in the status of women, prohibition against child marriage and expansion of education. The All India Women's Conference, founded in 1927 by Margaret Cousins for the upliftment and betterment of women and children, was an important example of the spreading consciousness among them.

Q4. Several foreigners made India their homeland and participated in various movements. Analyze their role in the Indian struggle for freedom. Ans: During the Indian National Movement, a number of foreigners played an important in the freedom struggle. Charles Freer Andrews, a British Christian missionary and social reformer came to India and later became a close friend of Mahatma Gandhi and identified completely with the cause of India's independence. Sister Nivedita (Margaret Elizabeth Noble) was a Scots-Irish social worker, author, teacher and a disciple of Swami Vivekananda. She had close associations with the Ramakrishna Mission but because of her active contribution in the field of Indian Nationalism, she had to publicly dissociate herself from the activities of the Ramakrishna Mission. A British, Anne Besant, who was the chief of the Theosophical Society in India, played a leading role during the Home Rule Movement and later was elected as the first woman President of the Indian National Congress. Another British, Nalini Sengupta, after her marriage to Jatindra Mohan Sengupta, became an eminent nationalist, who actually understood the contemporary Indian social and political scenario, in terms of the freedom struggle. Three British communist leaders, namely, Philip Spratt, Ben Bradley and Lester Hutchinson were accused under the Meerut Conspiracy Case in 1929. They were actively working for the left wing of the Indian National Movement. Q5. In many ways, Lord Dalhousie was the founder of modern India. Elaborate. Ans: Lord Dalhousie served as the Governor-General of India from 1848 to 1856. To promote British economic interests, he introduced a numbers of programmes, which also helped in the modernization of India. He also introduced social reforms against the social evils prevalent in India. He extended the British Rule in India through his doctrine of lapse and wars such as second Anglo-Sikh war. Thus, he united a larger part of India under similar rule. He introduced the development of infrastructure in India. He was a prominent supporter of the development of railways in India, which was introduced in 1853. He completed the Ganges canal and made liberal provisions for metalled roads and bridges. He created an imperial system of post-offices, reducing the rates of carrying letters and introducing postage stamps. He created the department of public instruction and inaugurated the system of administrative reports. He encouraged the culture of tea. He ensured protection of forests and preservation of ancient and historic monuments. He introduced the Widow Remarriage Act. He also introduced the Lex Loci Act- a uniform civil code that administered the same set of secular civil laws to govern all people irrespective of their religion, caste and tribe. Q6. Critically discuss the objectives of Bhoodan and Gramdan movements initiated by Acharya Vinoba Bhave and their success. Ans: Bhoodan and Gramdan movements were voluntary land reform movements in India started by Acharya Vinoba Bhave in 1951. It was to persuade wealthy landowners to voluntarily give a percentage of their land to the landless. However, this land could not be sold. The initial objective of the movement was to secure voluntary donations of land and distribute it to the landless, but the movement soon came out with a demand of 1/6th share of land from all land owners. In 1952, the movement had widened the concept of gramdan and had started advocating commercial ownership of land. But the movement could not get the expected success due to a number of reasons. The land obtained by landless people under the movement could not be sold. In effect, landless labourers were being given a small plot of land on which to settle, as well as grow some of their own food, so as to give them an incentive to remain in the village as a captive labour pool for the richer farmers and landlords. The land donated under the movement was often barren and infertile and it was much more costly to grow crops on it. The quantity of the land, received under the movement was very less and the number of the needy people was much more. Thus, none of the recipients could get a sufficient piece of land that could be used as the source of livelihood. Q7. Write a critical note on the evolution and significance of the slogan Jai Jawan Jai Kisan. Ans: The slogan of Jai Jawan Jai Kisan means hail the soldier and hail the farmer. Indian Prime Minister Lal Bahadur Shastri gave this slogan to India in 1965 when the country was attacked by Pakistan and at the same time was facing a scarcity of food grains. The slogan was given to enthuse the soldiers to defend India and simultaneously cheering farmers to do their best to increase the production of food grains to reduce dependence on import. The slogan inspired not only the soldiers and the farmers but it also motivated all the Indians to face the concurrent national emergency and to develop a sentiment of national unity. India repulsed the Pakistani invasion successfully. After Pokharan tests in 1998, Atal Bihari Vajpayee, the then Prime Minister, added Jai Vigyan (Hail knowledge) to the slogan. The slogan Jai Jawan Jai Kisan reverberates even today through the length and breadth of the country. Q8. Discuss the contribution of Maulana Abul Kalam Azad to pre and post independent India. Ans: Maulana Abul Kalam Azad, was born on November 11, 1888 in Mecca and was popularly known as Maulana Azad. He was one of the foremost leaders of Indian freedom struggle. He met Aurobinto Ghosh and Sri Shyam Shundar Chakravarty and joined the revolutionary movement against the British rule. He helped set up secret revolutionary centers all over north India and Bombay. In 1912, he started a weekly journal in Urdu called Al-Hilal to increase the revolutionary recruits amongst the Muslims. The British Indian government regarded Al- Hilal as the propagator of secessionist views and

www.careerlauncher.com/news/2013/UPSCmains2013GSanalysis.html

4/27

1/9/14

Civil Services Main Exam 2013


banned it in 1914. Then he started another weekly called Al-Balagh with the same mission of propagating Indian nationalism and revolutionary ideas based on Hindu-Muslim unity. In 1916, this weekly was also banned. He roused the Muslim community through the Khilafat Movement. He supported Non-Cooperation Movement and entered Indian National Congress in 1920. He was elected as the president of the special session of the Congress in Delhi in 1923. He was again arrested in 1930 for violation of the salt laws. He became the president of Congress in 1940 at Ramgarh and remained in the post till 1946. He was a staunch opponent of partition and supported a confederation of autonomous provinces. He was hurt greatly by the partition. He ws the first education minister in independent India and served as the Minister of Education in Pandit Jawaharlal Nehru's cabinet from 1947 to 1958. Q9. Analyse the circumstances that led to Tashkent Agreement in 1966. Discuss the highlights of the agreement. Ans: The Tashkent Agreement was a peace agreement between India and Pakistan after the Indo-Pakistan War of 1965. A number of reasons were liable for the execution of this agreement. Both United States of America and Union of Soviet Socialist Republics (Soviet Union) intervened in the matter and pushed Pakistan and India to a cease-fire in fear that the conflict might escalate and draw in other powers. India, having no objective to begin or to continue the war, agreed for the peace agreement. It did not want to lose the support of Soviet Union in the United Nations on the issue of Kashmir, and did not want to risk losing the supply of MIG fighter aircrafts, main battle tanks and other equipment from the Soviet Union. The Indian Army had broken the delusion of Pakistan that India had been weakened after its so-called defeat against China in 1962, and Pakistan could defeat India with more advanced weapons gained from United States. Furthermore, Pakistan was unable to continue the war because it was totally dependent on United States for its war efforts Continuation of the war could have caused unbearable damage of wealth and lives for both the countries. As per the agreement, both the countries agreed: - to pull back their forces to their pre-conflict positions. - to not interfere in each other's internal affairs. - to restore economic and diplomatic relations between them. - for orderly transfer of Prisoners of War. - to work towards building good relations between the two countries. Q10. Critically examine the compulsions, which prompted India to play decisive roles in the emergence of Bangladesh. Ans: Emergence of Bangladesh was the most important result of the Indo-Pak War, 1971. After independence from British Rule, the Western Pakistan (Pakistan) imposed its hegemony over the East Pakistan (now Bangladesh). East Pakistan revolted against the hegemony of Western Pakistan and later this revolt converted into the liberation war of Bangladesh. To escape from the brutality and genocide executed by Pakistan, a large number of refugees from Bangladesh entered into the border states of India and became an unbearable burden on India for their need of food and health services. The refugees constituted an exiled government in India and launched a liberation war against Pakistan. They received sympathy, help and assistance from the Indian people along with the Government of India on a large scale. Pakistan propagated this as a conspiracy of India against it and to attract the international attention on the entire issue, initiated a war against India. Pakistan also wanted to divert the attention of its own people from the liberation war towards the assumed conspiracy. India also wanted to end the matter of West Pakistan and East Pakistan and to have a friendly government for the people in East Pakistan. Q11. Latecomer Industrial revolution in Japan involved certain factors that were markedly different from what the West had experienced. Ans: The industrial revolution in Japan happened during the second half of the 19th century after Meiji Restoration . The industrial development of Japan was much more rapid than that of European nations. Foreign pressure from Britain, US, and French pushed Japan to open up its economy to outside nations and later this led to industrial revolution. Industrialization was seen as the solution to Japan's problems with outside nations. Imperial authorities pushed industrialization. Government hired foreign experts to show workers and managers how to modernize the economy. Japan had very little minerals which were required as basic materials, such as iron and coal, for industrial development an hence, was dependent on the import of essential minerals for its development on a large scale. Industrialization of Japan was followed by its rapid colonial expansion. Q12. Africa was chopped into states artificially created by accident of European competition. Analyse. Ans: Africa was colonized by European powers during 19th century till the 20th century, when every part of the continent was colonized at least once. The modern colonial period began ending in the 1950s and fully ended by the 1970s. A number of European powers such as Britain, France, Germany, Netherland, Belgium etc. made their colonies in Africa. The future of African states and communities was decided by European powers without involving African people in the process and they decided borderlines of African states according to their own needs and convenience. The borders of African states were decided over the map and were changed according to the negotiations of colonial needs among European powers. This is why African states do not have natural continental borders. This situation causes conflicts among African states even today. Q13. American Revolution was an economic revolt against mercantilism. Substantiate. Ans: The American Revolution was a struggle between the united thirteen colonies and Britain. The revolution ended in 1783 with the Treaty of Paris and the colonies won their independence and were known

www.careerlauncher.com/news/2013/UPSCmains2013GSanalysis.html

5/27

1/9/14

Civil Services Main Exam 2013


as United States of America. The major cause of the revolution was economic exploitation of colonies for interest of Britain and its mercantile hegemony over colonies. Proclamation of 1763, Sugar Act of 1764, Currency Act of 1764, Quartering Act, Stamp Act and many other steps were taken by the British Government to regulate and control the trade and commerce of colonies in the interest of the British trade and commerce. Through different acts and means, traders of colonies were being deprived from a more beneficial trade with any European nation other than Britain. Britain tried to keep them under its own control for Britains greater interests. The trade of colonies with other European nations could be operated only through Britain and only British ships could transport it. This system confined the profits that could be earned by free trade. The famous slogan of the revolution No Taxation Without Representation also indicates the importance of economic reasons liable for the revolution. Q14. What policy instruments were deployed to contain the great economic depression? Ans: The New Deal, a series of domestic economic programs between 1933 and 1936 was enacted in United States of America to tackle the great economic depression. The deal focused on the "3 Rs": Relief, Recovery, and Reform i.e. relief for the unemployed and poor; recovery of the economy to normal levels; and reform of the financial system to prevent a repeat depression. The depression was concerned with diverse groups, from banking and railroads to industry and farming, all of which demanded help for economic survival. The Federal Emergency Relief Administration, for instance, provided $500 million for relief operations executed by states and cities, while the short-lived Civil Works Administration gave money to localities to operate make-work projects in 1933-34. The deal included the Wagner Act to promote labor unions, the Works Progress Administration relief programme, the Social Security Act, and new programs to aid tenant farmers and migrant workers. The Tennessee Valley Project was a part of the programme. Under the deal, assistance was given to European nations also to handle the effects of the depression.

Q15. Discuss the various social problems which originated out of the speedy process of urbanization in India. Ans: As of 2011, 31.2 percent or 377 million population of India lived in urban areas. The urbanization of India is growing rapidly which has caused a number of social problems. Personal disorganization is the most important consequence of the growing urbanization. The urban life style promotes formal relations among the people and it increases insecurity in personal relations. Often people feel self alienated in the urban environment, which increases insensitivity towards others. Personal emotions and feelings are disorganized in a formal life. Growing urbanization has changed the role and contribution of family members. Often, earning couples fail to reconcile and coordinate with each other and it leads to separation of husband and wife. Children of such families are also not adequately cared for and they grow up with improper emotional development, which leads them towards personal disorganization. Social institutions, such as neighborhood, which regulate society and develop healthy social relations, are declining due to urbanization. It results in the decay of social and moral values. In such a condition, people try to fulfill their own wills and do not care for the greater welfare of the society. Urbanization prefers economic relations to social relations, which ultimately causes individual and social conflicts. Increasing extra-marital relations, underage sex, paid sex, suicidal cases, aggressive and impatient behavior of the people are the natural consequences of personal and social disorganization. Rapid urbanization has provided more space for these crimes. Q16. Male membership needs to be encouraged in order to make womens organization free from gender bias. Comment. Ans: It is very natural for an organization to be biased if its membership is confined to one gender. Absence of the other gender members hardly provides a chance to know about their views on an issue related to the welfare of the present gender members. Hence, womens organizations should also know about the approach of men on issues related to the empowerment and welfare of women. Mens immediate response on the issues can be noticed if they are members of the same organization. A better coordination can be created between men and women to successfully launch a mission or campaign for the welfare of women. Also, mens cooperation can be easily gained on an issue related to womens welfare. Q17. Critically examine the effects of globalization on the aged population in India. Ans: Globalization is the process of international integration arising from the interchange of world views, products, ideas, and other aspects of culture. According to a report of the United Nations Population Fund, the number of people with the age above-60s will increase to more than 300 million in India by 2050 and hence, there is a need to invest more in the health and welfare services. Due to globalization, some older people will be benefited, especially who are able to ensure their access to pensions and health care services, while poor men and women will suffer. Free trade, economic restructuring, the globalization of finance and the surge in migration will, in most parts of the world, tend to produce harmful consequences for older people. Globalization transfers resources from the poor to the rich within and between countries. It therefore increases social problems while simultaneously diminishing the freedom and capacity of India (like other countries) to make social policy. Nonetheless the effects of globalization and particularly its financial dimensions on India's capacity for making social policy can be exaggerated. Political will can combat international economic orthodoxy but the evident cases are the exception rather than the rule. Q18. Growing feeling of regionalism is an important factor in the generation of demand for a separate state. Discuss. Ans: According to the Home Ministry of India, it has received representations for creation of more than 20 states and if these demands are conceded, there will be at least 50 states in the country. Some examples are demands for Awadh Pradesh, Poorvanchal, Bundelkhand and Pachimanchal or Harit Pradesh in Uttar Pradesh, Kukiland in Manipur, Kongu Nadu in Tamil Nadu, Kamatapur in North Bengal, Tulu Nadu in

www.careerlauncher.com/news/2013/UPSCmains2013GSanalysis.html

6/27

1/9/14
Karnataka etc.

Civil Services Main Exam 2013


Demand for a separate state arises due to the regional discontent created by poor economic development. The ambition of the people of a region to brand their regional culture and identity on global level is also a reason for the demand of partition. It is also argued that smaller states can have a more responsive administration because more attention can be paid to a smaller population. Though it is not always true, especially if we see in reference to Uttarakhand, Jharkhand, Chhattisgarh etc. Political parties, ruling at state level, are often charged to implement development projects in a particular region of the state and it creates discontent in other regions, which feel ignored. Q19(a). What do you understand by the theory of continental drift? Discuss the prominent evidences in its support. Ans: Continental drift is the movement of the Earth's continents relative to each other by appearing to drift across the ocean bed. The theory of continental drift was superseded by the theory of plate tectonics, which explains how the continents move. Similar plant and animal fossils were found around different continent shores, suggesting that they were once joined. The complementary arrangement of the facing sides of South America and Africa is obvious, but is a temporary coincidence. In millions of years, slab pull and ridge-push, and other forces of tectonophysics will further separate and rotate those two continents. Widespread distribution of Permo-Carboniferous glacial sediments in South America, Africa, Madagascar, Arabia, India, Antarctica and Australia was one of the major pieces of evidence for the theory of continental drift. Q19(b) The recent cyclone on the east coast of India was called Phailin. How are the tropical cyclones named across the world? Elaborate. Ans: Tropical cyclones are named to provide ease of communication between forecasters and the general public regarding forecasts, watches, and warnings. There is no scientific rule to name cyclones. They are often named after the people (male or female), who are disliked.In 1945, the armed services of United States publicly adopted a list of women's names for typhoons of the western Pacific. The North Indian Ocean region tropical cyclones are named as of the arrangement of 2006. The Southwest Indian Ocean tropical cyclones were first named during the 1960/1961 season.

Q20 (a) Bring out the causes for the formation of heat islands in the urban habitat of the world. Ans: The urban heat island is a phenomenon in which central regions of urban centers exhibit higher mean temperatures than the surrounding urban areas. Much of this effect can be attributed to low city albedo, the reflecting power of a surface and the increased surface area of buildings that absorbs solar radiation. Concrete, cement, and metal surfaces in urban areas tend to absorb heat energy rather than reflect it, contributing to higher urban temperatures. The heat island effect has corresponding ecological consequences on resident species. However, this effect has only been seen in temperate climates. Q20(b) What do you understand by the phenomenon of temperature inversion in meteorology? How does it affect the weather and the habitants of the place? Ans: Temperature inversion is a reversal of the normal behaviour of temperature in the troposphere in which a layer of cool air at the surface is overlain by a layer of warmer air. Under normal conditions, air temperature usually decreases with height. Inversions play an important role in determining cloud forms, precipitation, and visibility. An inversion acts as a cap on the upward movement of air from the layers below. Inversions also affect diurnal variations in air temperature. Q21. Major hot deserts in northern hemisphere are located between 20-30 degree north and on the western side of the continents. Why? Ans: The main reasons for the location of the major hot deserts in the northern hemisphere are as under: 1. This is high air pressure region. 2. Mostly dry air currents are found here. 3. Some places are sheltered from rain-bearing winds from mountains. 4. Cold Ocean currents, often found at the western coast of the continents, are also an important reason. Q22(a) Bring out the causes for more frequent landslides in the Himalayas than in Western Ghats. Ans: Frequent landslides are a common problem in the Himalayas due to the geographical features of the mountains themselves. This relatively younger mountain belt is a geologically unstable area and is subject to more seismic movement. Landslides are more common during the rainy season when the soil is heavy with moisture or drainage is impeded. Volcanoes, earthquakes and other geological activities also cause landslides. Human activities related to agriculture, transportation, construction, mining etc. also increase the possibility of a landslide, whereas the geographical structure of Western Ghats provides it more stability. Q22 (b) There is no formation of deltas by rivers of the Western Ghats. Why? Ans: A river delta is a landform that is formed at the mouth of a river from the deposition of the sediment carried by the river as the flow leaves the mouth of the river. There is no formation of deltas by rivers of the Western Ghats because the slope of Western Ghats is sharp, whereas the margin of the plain along its side is narrow. Due to this, the rivers at Western Ghats cannot deposit sediments at their mouth and thus they can not create deltas. Q23 (a) Do you agree that there is a growing trend of opening new sugar mills in the Southern states of India? Discuss with justification. Ans: During the current session of 2013-14, a decline in the production of sugar in Southern India is expected. The major reason of this decline is the drought in Maharashtra and poor rains in Tamil Nadu. The plantation area of sugarcane in North Indian states has increased during the current session. During the last few years, a trend of opening new sugar mills has grown in the southern states of India. The reason behind this trend is the better quality of sugarcane grown in Southern India than that of North India, along with the long crush season of sugarcane. After 2000, ten new sugar mills were established in

www.careerlauncher.com/news/2013/UPSCmains2013GSanalysis.html

7/27

1/9/14
Tamil Nadu.

Civil Services Main Exam 2013

Q23 (b) Analyse the factors for highly decentralized cotton textile industry in India. Ans: 60% of the Indian textile Industry is cotton based. The cotton textile industry has been a traditional industry in India for centuries based on countrywide cottage and small scale industries. This industry is highly decentralized in India and the country has been one of the major producers of cotton cloths for centuries. Production of cotton and demand for cotton textiles in India is countrywide. Mostly every center of cotton textile industry has its own features and identification of its products. Adequate labour supply at relatively competitive wages at regional level, presence of integrated concept to consumer, efficient local entrepreneurial class etc. are some of the factors for the decentralization of the cotton textile industry. Presently, small scale producers, scattered at different centers of the country, are the largest contributors in the cotton textile industry. Q24. With growing scarcity of fossil fuels, the atomic energy is gaining more and more significance in India. Discuss the availability of raw material required for the generation of atomic energy in India and in the world. Ans: Uranium and Thorium are the main raw materials used for the generation of atomic energy. India is operating prototype reactors based on Thorium. Major producer countries of Uranium are Canada, United States, Russia, South Africa, Namibia, Niger, Brazil, Kazakhstan, Australia, France etc. Uranium reserves are found in a number of states of India such as Jharkhand, Chhattisgarh, Karnataka (Gogi), Maharashtra (Mogarra), Rajasthan (Umra, Rohil), Uttar Pradesh (Naktu), Uttarakhand (Pokharitunji) etc, India is the largest producer of Thorium in the world. Deposits of Monazite (source of Thorium) are found in India in many states such as Kerala Andhra Pradesh, Odisha, Chhattisgarh, Jharkhand, Gujarat, Rajasthan etc. Other countries that produce Thorium are United States, Canada, Brazil, South Africa, Malaysia, Australia etc. Q25. It is said that India has substantial reserves of shale oil and gas, which can feed the needs of the country for a quarter of the century. However, tapping of the resources doesnt appear to be high on the agenda. Critically discuss the availability and the issues involved. Ans: The US Energy Information Administration estimates Indias total reserve recoverable shale gas at 96 trillion cubic feet. In India, shale deposits are found across the Gangetic plain, Assam, Rajasthan and many coastal areas. India has recently approved the policy that allows exploration and exploitation of unconventional shale gas and oil on licenses that have already been awarded for conventional efforts. The policy will allow companies to apply for shale gas and oil rights in their petroleum exploration license and petroleum mining lease. Companies will be permitted three assessment phases, each with a maximum period of 3 years. Royalties and taxes for exploration done under this policy would be the same as for conventional production in a particular area. The environmental risks associated with the exploration and exploitation of shale gas and oil also have to be solved. The risks may include gas or dissolved minerals moving through other rocks into aquifers, leaks from production wells into neighbouring rock formations and aquifers, leaks of gas to the atmosphere, spills of fluids that come to the surface from storage tanks or lagoons. All these risks can be controlled through proper design and management of the drilling and extraction site. SOLUTION GENERAL STUDIES PAPER II Q1. The role of individual MPs (Members of Parliament) has diminished over the years and as a result healthy constructive debates on policy issues are not usually witnessed. How far can this be attributed to the anti-defection law, which was legislated but with a different intention? Ans: Virtually in every parliamentary democracy, it is crucial how each legislator votes on a particular bill. The vote of a legislator is supposed to be representative of his/her constituency. However, Indian MPs have often switched their allegiance at the time of any major vote without keeping in mind the constituencys views or the party manifesto. To prevent such defections/ horse-trading, major parties got together to amend the Constitution to this effect, not once but twice. The present provisions in the Constitution now disqualify legislators who switch allegiances and require them to vote as per the instructions of the party whip. It no longer matters what an individual legislator or his/her constituency thinks. It only matters what the party leadership requires him/her to think while voting. In addressing the horse-trading problem, anti-defection law has diminished the role of individual legislators and impacted various quarters of our democracy. As individuals, many MPs argue that the grip of the anti-defection law needs to be relaxed, that debates should be placed outside its purview. Analysts correctly stated that as duly elected representatives of the people, they retain the right to vote as they see fit, that law-making is Parliaments domain. It is an enlightened argument that, however, militates against the stifling of possible dissent or disagreement among individual MPs by the party whip. Q2. Discuss Section 66A of IT Act with reference to its alleged violation of Article 19 of the Constitution. Ans: It is alleged that Section 66A curbs freedom of speech and expression and violates Articles 14, 19 and 21 of the Constitution. Section 66A (a) refers to the sending of any information through a communication service that is grossly offensive or has menacing character. The meaning of the term grossly offensive in Section 66A (a) is crucial and remains undefined in India. Section 66A should not be considered a reasonable restriction within the meaning of Article 19 of the Constitution. Section 66A (b) is even more problematic than Section 66A (a) because it makes an offence of sending through a computer resource or communication device any information which he knows to be false, but for the purpose of causing annoyance, inconvenience, danger, obstruction, insult, injury, criminal intimidation, enmity, hatred or ill will, persistently by making use of such computer resource or a communication device. It is alleged that it cannot be a legitimate legislative objective to restrict freedom of speech in order to prevent annoyance or inconvenience. Causing insult or ill will or enmity could be a criminal offence if it amounts to defamation but can not be construed as a reasonable restriction within the meaning of Article 19 of the Constitution.

www.careerlauncher.com/news/2013/UPSCmains2013GSanalysis.html

8/27

1/9/14

Civil Services Main Exam 2013


Q3. Recent directives from Ministry of Petroleum and Natural Gas are perceived by the `Nagas as a threat to override the exceptional status enjoyed by the State. Discuss in light of Article 371A of the Indian Constitution. Ans: The Centre and Nagaland government had landed in a constitutional row over the state's right to independently exploit its natural resources. The Centre objected to the issuance of Nagaland Petroleum and Natural Gas Regulations by the state government saying that the state, under the Constitution, could not regulate mining of natural wealth under the ground as this was a Central subject. It argued that Article 371A only gives the state, the right against implementing Central Acts in certain areas and not to make new laws or regulations on these subjects. The state government, however insisted that Article 371A makes it clear that no Act of Parliament in respect of Naga customary laws, administration of civil and criminal justice involving decisions according to customary law, ownership and transfer of land and its resources shall apply to the state of Nagaland unless the Legislative Assembly of Nagaland by a resolution so decides. Legally Article 371A of the constitution provides the state negative power as in the right to reject a Parliamentary Act in any of the areas mentioned in the provision. However it does not provide the state government the positive power to legislate or regulate aspects of the areas that may fall under Central control. Q4. The Supreme Court of India keeps a check on arbitrary power of the Parliament in amending the Constitution. Discuss critically. Ans: Supreme Court has been designated as the highest court of the country and the guardian of the constitution. The constitution confers a duty on the Supreme Court (SC) to protect the basic meaning and ideas enshrined in the constitution of India. And SC being the highest level of judiciary becomes the real guard behind this responsibility. There has been a conflict between the Supreme Court and Parliament, where Parliament wants to exercise discretionary use of power to amend the constitution while the Supreme Court wants to restrict that power. This has led to the laying down of various doctrines or rules in regards to checking the validity/legality of an amendment, the most famous among them is the Doctrine of Basic structure as laid down by the Supreme Court in the case of Kesavananda Bharati case. Q5. Many State Governments further bifurcate geographical administrative areas like Districts and Talukas for better governance. In light of the above, can it also be justified that more number of smaller States would bring in effective governance at State level? Discuss. Ans: State Governments further bifurcate geographical administrative areas like Districts and Talukas for better governance. This is more related to administrative convenience. But division of states has wider implications on socio-economic and political development of the state. A new state has to build the entire basic infrastructure which a state should have. Both large and small states will continue to be badly governed until there is effective devolution of funds, functions and functionaries to local authorities, that is, elected panchayats in villages and urban local bodies in cities. Arguments in favour of smaller states Factual analysis shows the development and efficiency argument does work in favour of the new states when compared with the parent states. During the tenth five-year plan period, Chhattisgarh averaged 9.2 percent growth annually compared with 4.3 percent by Madhya Pradesh, Jharkhand averaged 11.1 per cent annually compared with 4.7 percent by Bihar, and Uttarakhand achieved 8.8 per cent growth annually compared with 4.6 percent by Uttar Pradesh. Comparatively smaller but compact geographical entities tend to ensure that there is better democratic governance, as there is greater awareness among the policy makers about the local needs. Smaller states provide gains for the electorates in terms of better representation of their preferences in the composition of the government. In a patronage-based democracy like in India, the amount of the transfer of state resources or largesse, a constituency or region gets depends crucially on whether the local representative belongs to the ruling party. Arguments against smaller states Many fear the rise of regional and linguistic fanaticism as threats to national unity and integrity. Smaller states are often viewed as being much more vulnerable to the pressures of the corporations and multi-nationals due to their small scale economies and the greed of the newly emergent regional elite. Q6. Constitutional mechanisms to resolve the inter-state water disputes have failed to address and solve the problems. Is the failure due to structural or process inadequacy or both? Discuss. Ans: Interstate water disputes in India often prolong over long periods and tend to recur. The Cauvery dispute tribunal was constituted in 1990 and the final award was given in 2007, after 17 years. The 2nd Krishna water dispute tribunal, constituted in 2004, gave its final award recently in December 2010. These long delays are partly due to elaborate judicial proceedings and deliberations. The random nature of politics reshapes the nature and extent of a dispute and contributes to its frequent recurrence. Discourses of policy and governance reforms usually do not account for this contingent nature of politics. The frequent recurrence and long deliberations produce various kinds of insecurities and impact people's livelihoods. Indeed, the interstate water disputes in India have been on rise in recent years. Two other disputes under consideration are the Mullaperiyar (between Kerala and Tamil Nadu) and the Bhabli (between Maharashtra and Andhra Pradesh). These disputes caused concerns about their potential impact over State-State relations in India, with greater implications to the federal integrity of the nation-state. Adjudication by tribunals between states has remained inert to these multi-actor driven multi-scalar politics. The multiplicity of actors and processes constantly rework the dynamics of interstate water disputes, which in turn affect the emergence and recurrence of disputes. Q7. Discuss the recommendations of the 13th Finance Commission which have been a departure from the previous commissions for strengthening the local government finances. Ans: The recommendations of the 13th Finance Commission which have been a departure from the previous

www.careerlauncher.com/news/2013/UPSCmains2013GSanalysis.html

9/27

1/9/14

Civil Services Main Exam 2013


commissions for strengthening the local government finances are: Article 280 (3) (bb) & (c) of the Constitution should be amended such that the words on the basis of the recommendations of the Finance Commission of the State are changed to after taking into consideration the recommendations of the Finance Commission of the State. Article 243(I) of the Constitution should be amended to include the phrase or earlier after the words every fifth year. The states should appropriately allocate a portion of their share of the general basic grant and general performance grant, to the special areas in proportion to the population of these areas. This allocation will be in addition to the special area basic grant and special area performance grant recommended by Finance Commission. State Governments should appropriately strengthen their local fund audit departments through capacity building as well as personnel augmentation. The State Governments should incentivize revenue collection by local bodies through methods such as mandating some or all local taxes as obligatory at non-zero rates of levy. To buttress the accounting system, the finance accounts should include a separate statement indicating head-wise details of actual expenditures under the same heads as used in the budget for both Panchayati Raj Institutions (PRIs) and Urban Local Bodies (ULBs). The Government of India and the State Governments should issue executive instructions so that their respective departments pay appropriate service charges to local bodies. Given the increasing income of State Governments from royalties, they should share a portion of this income with those local bodies in whose jurisdiction such income arises. State Governments should ensure that the recommendations of State Finance Commissions (SFCs) are implemented without delay and that the Action Taken Report (ATR) is promptly placed before the legislature. Bodies similar to the SFC should be set up in states which are not covered by Part IX of the Constitution. Local bodies should consider implementing the identified best practices. A portion of the grants provided by Finance Commission to urban local bodies should be used to revamp the fire services within their jurisdiction. Local Bodies should be associated with city planning functions wherever other development authorities are mandated this function. These authorities should also share their revenues with local bodies. The development plans for civilian areas within the cantonment areas (excluding areas under the active control of the forces) should be brought before the district planning committees.

Q8. The product diversification of financial institutions and insurance companies, resulting in overlapping of products and services strengthens the case for the merger of the two regulatory agencies, namely SEBI and IRDA. Justify. Ans: Financial Sector Legislative Reforms Commission suggested that financial sector regulators such as SEBI and IRDA should be merged into a Unified Financial Agency (UFA). The Unified Financial Authority should aim to implement the consumer protection provisions and micro-prudential provisions for the entire financial system which can bring clarity in the minds of the investors. At present, in India, there is a confusing situation with regulators utilizing many instruments such as regulations, guidelines, circulars, letters, notices and press releases which at times create confusion in the minds of investors. In the coming 25 to 30 years, Indian GDP is estimated to become eight times larger than the present level and is likely to be bigger than the United States GDP as of today. So the need of the day is to create an institution that will stand the test of time. It is proposed to create another institution called FRA (Financial Redress Agency) which will be implementing this financial regulatory architecture. It will set up a nationwide machinery to become a one-stop shop where consumers can file complaints against all financial firms. Q9. The concept of Mid-Day Meal (MDM) scheme is almost a century old in India with early beginnings in Madras Presidency in pre-independent India. The scheme has again been given impetus in most states in the last two decades. Critically examine its twin objectives, latest mandates and success. Ans: The roots of the programme can be traced back to Pre-Independence era when a Mid Day Meal Programme was introduced by British administration for disadvantaged children in Madras Municipal Corporation in 1925. The primary objective of the scheme is to provide hot cooked meal to children of primary and upper primary classes and the other objectives are improving nutritional status of children, encouraging poor children belonging to disadvantaged sections to attend school more regularly and help them concentrate on classroom activities, thereby increasing the enrollment, retention and attendance rates. While the scheme mandates to provide for a hot cooked meal to ensure an energy content of 450 calories and 12 grams of proteins for children studying in primary classes and 700 calories of energy and 20 grams of proteins at the upper primary level, the Ministry of Human Resource Development has confirmed that 95 per cent of meal samples prepared by NGOs in Delhi did not meet nutritional standards in 2010-12. Similar trends can be seen in other states. Despite the success of the program, child hunger as a problem persists in India. According to current statistics, 42.5% of the children under 5 are underweight. The problem of malnutrition, anaemia, deficiency in vitamin A and iodine is very common among children in India. Q10. Pressure group politics is sometimes seen as the informal face of politics. With regard to the above, assess the structure and functioning of pressure groups in India. Ans: Pressure groups are the interest groups which work to secure certain interest by influencing the public policy. They are non-aligned with any political party and work as indirect yet powerful group to influence the decision. There is no formal structure of pressure groups but there are certain types of pressure groups like business groups, trade unions, professional associations which do have some formal structure. On other hand, there are anomic groups which are a response to an immediate issue or concern like riots, demonstrations, assassinations etc. and are more or less spontaneous in nature. In such groups, there is practically no formal structure. So the structure of pressure groups varies from situation to situation. Pressure group is the public body acting behind the political party (outside political party). Pressure group

www.careerlauncher.com/news/2013/UPSCmains2013GSanalysis.html

10/27

1/9/14

Civil Services Main Exam 2013


act is indirect as well as intermittent. They try to influence and pressurize the government to get their demand fulfilled. They do not intervene directly. Pressure groups pressurize executive and legislature to achieve its aim. They use both conventional and non-conventional means to demonstrate their demands. They work for self interest; they emerge and dissolve as per the need of certain groups. Q11. The legitimacy and accountability of Self Help Groups (SHGs) and their patrons, the micro-finance outfits, need systematic assessment and scrutiny for the sustained success of the concept. Discuss. Ans: A self-help group (SHG) is a village-based financial intermediary usually composed of local women or men. Members make small regular savings contributions over a few months until there is enough capital in the group to begin lending. Funds may then be lent back to the members or to others in the village for any purpose. In India, many SHGs are 'linked' to banks for the delivery of microcredit. But there are controversies in recovering patterns of micro-finance institutions (MFIs)and they are charging exorbitant interest rates which are directly impacting the poor. So, systematic assessment and scrutiny are needed in case of MFIs. Recent regulations passed by RBI MFIs must be adhered to. First, the borrowers in the Microfinance sector represent a particularly vulnerable section of society. They lack individual bargaining power, have inadequate financial literacy and live in an environment which is fragile and exposed to external shocks which they are ill-equipped to absorb. They can, therefore, be easily exploited. Therefore, MFIs need to be more cautious in its approach. Q12. The Central Government frequently complains on the poor performance of the State Governments in eradicating suffering of the vulnerable sections of the society. Restructuring of Centrally sponsored schemes across sectors for ameliorating the cause of vulnerable sections of population, aims at providing flexibility, in better implementation, to the States. Critically evaluate. Ans: Restructuring of Centrally sponsored schemes aims at following steps to be taken by central government to ensure better implementation at the state level: After taking into account the available technology and infrastructure for electronic flow of information and funds, especially under the NeGP and putting in place a new Chart of Accounts, the scheme should be implemented in a time bound manner. There should be provisions of untied funds in case of centrally sponsored schemes. A robust financial information system needs to be created in the government in a time bound manner. This system should make the real time data on government expenditure at all levels, accessible to the public, The Controller General of Accounts, in consultation with the C&AG, should lay down the principles for implementing the system of flow of sanctions or approvals from the Union Ministries or Departments, to the implementing agencies in the States, to facilitate release of fund at the time of payment. But the lack of implementation of the above steps is one of the biggest hindrances in not being able to realize the stated goals. Q13. Electronic cash transfer system for the welfare schemes is an ambitious project to minimize corruption, eliminate wastage and facilitate reforms. Comment. Ans: In electronic cash transfers, the money is directly transferred into bank accounts of beneficiaries. LPG and kerosene subsidies, pension payments, scholarships, employment guarantee scheme payments as well as benefits under other government welfare programmes will be made directly to beneficiaries under the electronic cash transfer scheme. It helps in minimizing corruption, eliminate wastage and facilitate reforms. It can help the government reach out to identified beneficiaries and can plug leakages. Like in case of PDS scheme, some ration shop owners divert subsidized PDS grains or kerosene to open market and make fast money. Such leakages can be stopped by the implementation of the electronic cash transfer scheme. The scheme will also enhance efficiency of welfare schemes. Moreover, electronic cash transfers also helps in furthering the process of e-governance. The Government spends a huge amount of Rs.3, 25,000 crore annually on subsidies and the new scheme is intended to check corruption and pilferage of subsidised items including diesel, LPG and leakages in other schemes like MNREGA, pension and scholarships. Also by plugging leakages, the total burden of subsidies can be reduced in the long run. Q14. The basis of providing urban amenities in rural areas (PURA) is rooted in establishing connectivity. Comment. Ans: Provision of Urban Amenities to Rural Areas (PURA) is a strategy for rural development in India. This concept was given by former president Dr. A.P.J. Abdul Kalam. PURA proposes that urban infrastructure and services be provided in rural hubs to create economic opportunities outside of cities. It aims at connecting rural hubs with urban areas. There are mainly three types of connectivity to be achieved. Connectivity: Physical connectivity by providing roads Electronic connectivity by providing communication network Knowledge connectivity by establishing professional and technical institutions Q15. Identify the Millennium Development Goals (MDGs) that are related to health. Discuss the success of the actions taken by the Government for achieving the same. Ans: Millennium Development Goals (MDGs) related to health are:To eradicate extreme poverty and hunger To reduce child mortality rates To improve maternal health To combat HIV/AIDS, malaria, and other diseases To ensure environmental sustainability Broadly, government is not likely to achieve the targets set by MDGs in reducing child mortality rates and maternal mortality rates. Poverty rates are reducing though targets set by MDGs are hard to achieve by 2015. For environment sustainability, National Action Plan on Climate Change was started which includes National Solar Mission, National Mission for Enhanced Energy Efficiency, National Mission on Sustainable Habitat National Water Mission, National Mission for Sustaining the Himalayan Ecosystem, National Mission for a Green India, National Mission for Sustainable Agriculture and National Mission on Strategic Knowledge for Climate Change. The government has taken various steps in this regard but still there is long way to go

www.careerlauncher.com/news/2013/UPSCmains2013GSanalysis.html

11/27

1/9/14

Civil Services Main Exam 2013


to achieve environmental sustainability. Q16. Though Citizens charters have been formulated by many public service delivery organizations, there is no corresponding improvement in the level of citizens satisfaction and quality of services being provided. Analyze. Ans: Many states and public service delivery organizations have enacted their own grievance redressal laws. The mechanism provided for grievance redressal results in multiplicity of work and organizations. Absence of lokpal/lokayukta or any related effective institution makes the exercise less meaningful. Many of these Citizens charters do not describe the sharing of responsibility in many cases. Under various state service acts, the commissioners may be removed without judicial inquiry, which makes it susceptible to misuse. No or very less help of NGOs is being taken in collecting feedback of citizens satisfaction. There are further no improvements regarding grievances. Many of these Citizens charters are exclusively for citizens. Q17. A national Lokpal, however strong it may be, cannot resolve the problems of immorality in public affairs. Discuss. Ans: Corruption or immorality in public affairs is recognized as the single biggest problem faced by the country today. Corruption has serious adverse effects on the society and the economy and it corrodes the moral fiber of the people. Corruption is a symbol of something gone wrong in the management of the state. But a national Lokpal, however strong it may be, cannot resolve the problems of immorality in public affairs. The mechanism to enforce accountability in the public affairs need not be the same for everybody. The exact judicial powers of Lokpal are rather unclear in comparison with its investigative powers. The critics hence express concern that, without judicial review, Lokpal could potentially become an extraconstitutional body with investigative and judicial powers whose decisions cannot be reviewed in regular courts. It has been drafted on the presumption that an act of corruption will be committed only by a Government Servant. This is evident from the partial inclusion of business entities and the total exclusion of NGOs, media and other similar categories of institutions from the Lokpals ambit. Proposed Lokpal in India recommends a maximum punishment of life imprisonment for corrupt public servants, while the punishment for a Lokpal member found guilty of corruption is just dismissal, removal or reduction in rank. Q18. The proposed withdrawal of International Security Assistance Force (ISAF) from Afghanistan in 2014 is fraught with major security implications for the countries of the region. Examine in light of the fact that India is faced with a plethora of challenges and needs to safeguard its own strategic interests. Ans: As Western combat troops prepare to leave the Afghanistan, some ISAF members and Asian regional players are stepping up their roles to achieve previously strived for goals of a stable, peaceful and selfreliant Afghanistan and maintain or gain influence in the region. There are major security implications and challenges faced by India to safeguard its own strategic interests. India does not want revival of extremist Taliban. Indias energy interests in Iran, Turkmenistan can be protected only with a stable and democratic Afghanistan. Friendly Afghanistan can also help in containing Pakistan. India seeks to expand its economic presence in Afghanistan as the international coalition fighting the Taliban withdraws combat forces through 2014. Especially, it wants to improve transport connectivity and economic collaboration with countries in Central and South Asia. The Indian government is investing more than US$100 million in the expansion of the Chabahar port in south-eastern Iran which will serve as a hub for the transportation of transit goods. Q19. What do you understand by The String of Pearls? How does it impact India? Briefly outline the steps taken by India to counter this. Ans: The String of Pearls refers to the network of Chinese military and commercial facilities, and the relationships along its sea lines of communication, which extend from the Chinese mainland to Port Sudan. Exercise Malabar is a multilateral naval exercise involving the United States, India, Japan, Australia, and Singapore. India's 'Look East' policy with the south-east Asian countries is also considered as a broad strategy to counter the String of Pearls. As the current chair of ASEAN, Vietnam had also invited India to the ASEAN+8 defence ministers meeting. The newly evolving India-South Korea partnership is being seen as a vital component of India's game plan to counter China's increasing footprint in the Indian subcontinent.

Q20. Economic ties between India and Japan while growing in the recent years are still far below their potential. Elucidate the policy constraints which are inhibiting this growth. Ans: Japan is currently Indias third largest source of foreign direct investment. Japan has assisted India in infrastructure development projects such as the Delhi Metro Rail Project. Both sides are discussing the Delhi-Mumbai Industrial Corridor Project and Dedicated Freight Corridor Projects on the Mumbai-Delhi and the Delhi-Howrah routes. The Japanese government has also expressed interest to help establish a Chennai-Bangalore Industrial corridor and a Dedicated Freight project in the south, connecting the cities of Bangalore and Chennai. Moreover, the bilateral trade at $18 billion between the two countries is not very impressive and leaves much to be desired especially when the bilateral trade between India and China has crossed $70 billion. Both countries stand to gain from their strategic partnership but there are still some unresolved issues in India-Japan relations. There is considerable scope for increased Japanese investment in India as it is much below its potential. Japanese companies have been conservative while dealing with India. Q21. The protests in Shahbag Square in Dhaka in Bangladesh reveal a fundamental split in society between the nationalists and Islamic forces. What is its significance for India? Ans: The 2013 Shahbag protests, associated with a central neighbourhood of Dhaka, later spread to other parts of Bangladesh as people demanded capital punishment for Abdul Quader Mollah who had been sentenced to life imprisonment and for others convicted of war crimes by the International Crimes Tribunal.

www.careerlauncher.com/news/2013/UPSCmains2013GSanalysis.html

12/27

1/9/14

Civil Services Main Exam 2013


On that day, the International Crimes Tribunal had sentenced Abdul Quader Mollah to life in prison after he was convicted on five of six counts of war crimes. Later, demands included banning the Bangladesh Jamaate-Islami party from politics and a boycott of institutions supporting (or affiliated with) the party. Finally in September 2013, Bangladesh Supreme Court found Abdul Quader Molla guilty of murders and other war crimes and ordered his execution, converting his life sentence to death sentence. Q22. Discuss the political developments in Maldives in the last two years. Should they be of any cause of concern to India? Ans: The Maldives political crisis began as a series of peaceful protests that broke out in the Maldives in May 2011. They continued, eventually escalating into the resignation of President Mohamed Nasheed in disputed circumstances in February 2012. Nasheed stated the next day that he was forced out of office at gunpoint, while Waheeds (the next President) supporters maintained that the transfer of power was voluntary and constitutional. After Nasheed and his supporters continued to protest for Waheed's removal from office, he agreed to call a snap election. Presidential elections were held in the Maldives under a two-round system. The result of the initial vote held was annulled by the Supreme Court and the election was re-run in November. As no candidate achieved majority support, a run-off election was held. In the run-off elections between Mohamed Nasheed and Abdullah Yaamin, who was a conservative candidate related to former autocratic ruler Maumoon Abdul Gayoom, Yaamin was elected as the new president of the Maldives. India always maintained that it will not interfere in the internal affairs of another country. India wants democracy to be propagated among its neighbours so that it can take on the path of stability, progress and development. India, which sees the Maldives as part of its sphere of influence and has a very strong strategic interest in the island nation, is sure to be relieved after the elections. Political uncertainty has been a matter of concern in this crucial archipelago. India does not want the island nation to become a bastion of political Islam and religious extremism. India is also wary of growing Chinese interests in the island nation. Q23. In respect of India Sri Lanka relations, discuss how domestic factors influence foreign policy. Ans: Political leaders of almost all hues in Tamil Nadu have called for a boycott of the Commonwealth Heads of Government Meeting (CHOGM). The issue of Sri Lankan Tamils played a role in Tamil politics as seen in the state elections. Tamil politics consider the Centres response to Sri Lankas war crimes and human rights aberrations as inadequate and ineffective. The developments in Tamil Nadu seem to have influenced Indias vote against Sri Lanka in the United Nations High Commissioner for Refugees (UNHCR). The Northern Provincial Council election was held due to Indias sustained engagement with Sri Lanka. But Indias relationship with its island neighbour goes beyond the geo-strategy and Tamil minority question. Geo-strategically, India wields a huge influence over the Sri Lanka. With China whittling away Indias strategic sphere of influence in the South Asian neighbourhood, India has to consider not only its national interest but also the regional interest while taking decisions that affect its neighbours. And in Sri Lanka, China is emerging as a direct challenge to Indian presence. In fact, China effectively used the aftermath of the Eelam war to emerge as one of the two big investors and aid-givers in Sri Lanka, the other being India. Q24. What is meant by Gujral doctrine? Does it have any relevance today? Discuss. Ans: The Gujral Doctrine is a set of five principles to guide the conduct of foreign relations with Indias immediate neighbours, notably Pakistan, as spelt out by Gujral. 1. With neighbours like Nepal, Bangladesh, Bhutan, Maldives and Sri Lanka, India does not ask for reciprocity but gives all that it can in good faith and trust. 2. No South Asian country will allow its territory to be used against the interest of another country of the region. 3. None will interfere in the internal affairs of another. 4. All South Asian countries must respect each others territorial integrity and sovereignty. 5. They will settle all their disputes through peaceful bilateral negotiations. In the context of the changed international environment in post-cold war world, Gujral Doctrine became a new and important principle of India's foreign policy. Sharing of Ganga Water with Bangladesh: It is in pursuance of this policy that India concluded an agreement with Bangladesh in late 1996, on sharing of Ganga waters. This agreement enabled Bangladesh to draw slightly more water, in the lean season, than even the 1977 Agreement had provided. Gujral doctrine helps in strengthening Track-II diplomacy by focusing on increasing People to People Contact with Pakistan, Confidence Building Measures so as to build confidence and seek friendly resolution of all disputes. Q25. The World Bank and the IMF, collectively known as the Bretton Woods Institutions, are the two inter-governmental pillars supporting the structure of the worlds economic and financial order. Superficially, the World Bank and the IMF exhibit many common characteristics, yet their role, functions and mandate are distinctly different. Elucidate. Ans: The World Bank is an international financial institution that provides loans to developing countries for development programs. The World Bank's official goal is reduction of poverty. It aims to promote foreign investment and international trade and the facilitation of capital investment. The World Bank comprises two institutions: the International Bank for Reconstruction and Development (IBRD) and the International Development Association (IDA). For the poorest developing countries in the world, the bank's assistance plans are based on poverty reduction strategies; by combining a cross-section of local groups with an extensive analysis of the country's financial and economic situation, the World Bank develops a strategy pertaining uniquely to the country in question. The government then identifies the country's priorities and targets for the reduction of poverty, and the World Bank aligns its aid efforts correspondingly. The IMF's stated goal was to assist in the reconstruction of the world's international payment system post World War II. The organization's stated objectives are to promote international economic co-operation, international trade, employment and exchange rate stability, including by making financial resources available to member countries to meet balance of payments needs.

www.careerlauncher.com/news/2013/UPSCmains2013GSanalysis.html

13/27

1/9/14

Civil Services Main Exam 2013


However, many of the new roles of the IMF are inconsistent with the original mandate. The fund's function as lender of short-term finance was meant to fix balance of payments problems, but has expanded so much that the IMF can now be seen as an aid agency. Specifically the Poverty Reduction and Growth Facility (PRGF), the former Enhanced Structural Adjustment Facility (ESAF), allows the IMF to essentially subsidize interest rates for the poorest member countries.

SOLUTION GENERAL STUDIES PAPER III

1. With a consideration towards the strategy of inclusive growth, the new Companies Bill, 2013 has indirectly made CSR a mandatory obligation. Discuss the challenges expected in its implementation in right earnest. Also discuss other provisions in the Bill and their implications. Ans. The new Companies Bill has made it mandatory for profit making companies to spend on activities related to the Corporate Social Responsibility (CSR). With the new legislation, India would possibly become the first country to have the Corporate Social Responsibility (CSR) spending through a statutory provision. Following are the key highlights of the Bill: Around 193 recommendations have been included in the Companies Bill by the Parliamentary Standing Committee and with the passing of this Bill, the Companies Act of 1956 will be replaced. The proposed legislation would ensure setting up of special courts for speedy trials, stronger steps for transparent corporate governance practices and that corporate misdoings be curbed. The new law would require companies that meet certain set of criteria, to spend at least two percent of their average profits in the last three years towards the Corporate Social Responsibility (CSR) activities. But only companies reporting Rs 5 crore or more profits in the last three years have to spend on the CSR activities. At least 2% of the average net profit of the company made during three previous financial years must be spent on the CSR activities. Challenges: The Indian Merchants Chamber (IMC) has raised concerns about specific clauses in the new Companies Bill 2013. The (negative) repercussions of certain provisions of the bill must be considered. For instance, the definition and role of independent directors is raising concerns within the corporate sector. A sense of wariness and unease has crept in with the new code of accountability laid out in the Bill. Another bone of contention is related to the Corporate Social Responsibility (CSR). 2. What were the reasons for the introduction of Fiscal Responsibility and Budget Management (FRBM) Act, 2013? Discuss critically its salient features and their effectiveness. Ans. The Fiscal Responsibility and Budget Management (FRBM) Act, 2003 was enacted with a view to provide a legislative framework for reduction of deficit, and thereby debt, of the Government to sustainable levels over a medium term so as to ensure inter-generational equity in fiscal management and long term macro-economic stability. The FRBM framework provided a medium term perspective to fiscal management. The FRBM framework requires the Government to reduce the deficits to a prescribed target in a prescribed time following a laid out fiscal consolidation roadmap. It requires Government to place a medium terms fiscal framework laying down the projected fiscal aggregates to meet the fiscal targets as prescribed in Act/Rules. It also mandates the Government to spell out the strategy that it decides to adopt to meet the projects fiscal plan. Salient features: The Bill is also likely to reaffirm the medium-term fiscal targets that were proposed in Budget 2011-12. While in 2011-12 fiscal deficit was projected at 4.6 per cent of the GDP, it was planned that it will be brought down to 4.1 per cent in 2012-13 and then to 3.5 per cent in 2013-14. Likewise, revenue deficit was expected to be cut to 3.4 per cent in 2011-12, and then to 2.7 per cent in the subsequent fiscal and finally to 2.1 per cent in 2013-14.

3. What is the meaning of the term tax expenditure? Taking housing sector as an example, discuss how it influences the budgetary policies of the government. Ans. Tax Expenditure under the Central Tax System: The main objective of any tax system is to raise revenues necessary to fund government expenditures. The amount of revenue raised is determined to a large extent by tax bases and tax rates. It is also a function of a range of measures - special tax rates, exemptions, deductions, rebates, deferrals and credits - that affect the level and distribution of tax. These measures are sometimes called "tax preferences". They have an impact on government revenue (i.e. they have a cost) and reflect the policy choices of the Government. Section 80-IA of the Income-tax Act, 1961 provides for deduction in respect of profits derived from development of infrastructure facilities, SEZs and Industrial Parks, generation of power, and providing telecommunication services. Similarly, section 80-IB of the Income-tax Act, 1961 provides for deduction in respect of profits derived from housing projects, production of mineral oil, development of scientific research, integrated business of handling, storage and transportation of food grains, and industries set up in backward areas. The revenue foregone on account of these tax benefits has been estimated separately by adopting the proportionate assignment method described above. The same method has been adopted for estimating revenue foregone on account of section 80-I C. 4. Food Security Bill is expected to eliminate hunger and malnutrition in India. Critically discuss various apprehensions in its effective implementation along with the concerns it has generated in WTO. Ans: The Indian National Food Security Act, 2013 or Right to Food Act was enacted into law on September 12, 2013. This law aims to provide subsidized food grains to approximately two thirds of India's 1.2 billion people. Under the provisions of the bill, beneficiaries are to be able to purchase 5 kilograms of cereals per month for every eligible person at the following prices:

www.careerlauncher.com/news/2013/UPSCmains2013GSanalysis.html

14/27

1/9/14

Civil Services Main Exam 2013


Rice at INR 3 per kg, wheat at INR 2 per kg, coarse grains (millet) at INR 1 per kg. Pregnant women, lactating mothers, and certain categories of children are eligible for free meals daily. The bill has been highly controversial. It was introduced into India's parliament in December 2012, promulgated as a presidential ordinance in July 2013, and enacted into law in August 2013. Criticism: Food prices are still elevated and the food security bill will aggravate food price inflation as it will tilt supply towards cereals and away from other farm produce (proteins), which will further raise food prices. The food security bill, if implemented honestly, will cost 3 per cent of the GDP in its very first year. The governments estimated cost of food security comes at 11.10% of the total receipts. The CACPs estimated cost of food security comes at 21.5% of the total receipts. The Indian Ministry of Agriculture's Commission on Agricultural Costs and Prices warned that enactment of the Bill could be expected to "induce severe imbalance in the production of oilseeds and pulses and it will create demand pressures, which will inevitably spillover to market prices of food grains. 5. What are the different types of agriculture subsidies given to farmers at the national and state level? Critically analyse the agricultural subsidy regime with reference to the distortions created by it. Ans: Agricultural subsidies are financial support given to farmers to support their operations. Subsidies may be provided directly, in the form of cash payments, or they may take the form of indirect support. In India, at present, centre as well as state governments are providing subsidies on fertilizers, irrigation (canal water), electricity and other subsidies to marginal farmers and farmers cooperative societies in the form of seeds, development of oil seeds, pulses, cotton, rice, maize and crop insurance schemes and price support schemes etc. Out of these subsidies, the Central Government of India has been providing indirect subsidies to farmers on the purchase of fertilizers since 1977, whereas state governments are providing subsidies on irrigation as well as electricity Criticism: Subsidies are often criticized for their financial burden. Some researchers assert to the extent that these should be withdrawn in a phased manner. Such a step will reduce the fiscal deficit, improve the efficiency of resources used and fund for public investment in agriculture. On the other hand, there is a fear that agricultural production and income of farmers would decline if subsidies are curtailed. These are very important issues, which need serious investigation. Subsidies are often criticized for their financial burden. The objectives of the present study are to study the growth and distribution of agricultural subsidies in India, to study the impact of agricultural subsidies in India and to suggest ways and means for giving agricultural subsidies to farmers of India. 6. India needs to strengthen measures to promote the pink revolution in food industry for ensuring better nutrition and health. Critically elucidate the statement. Ans: India has already seen the green and white revolutions in its food industry come to pass; it now seems well on its way to realizing a pink revolution too; the modernization of meat production processes. Pink revolution and food security: The Food and Agriculture Organization of the UN (FAO) has now found that India is well on its way to modernizing meat and poultry processing, thus realizing the pink revolution too. In a report titled the Indian Meat Industry Perspective, the FAO outlined four steps that should be taken if Indias food industry is to successfully go pink. These recommended steps were: setting up state of the art meat processing plants; developing technologies to raise male buffalo calves for meat production; increasing the number of farmers rearing buffalo under contractual farming; and establishing disease-free zones for rearing animals. India has already become quite rosy and meat production has been steadily growing over the past decade. According to the United States Department of Agriculture (USDA) Foreign Agricultural Service, India became the largest exporter of buffalo meat in 2012, exporting approximately 1.5 million metric tons of beef. The largest importers of Indian meat are primarily countries in the Middle East and South East Asia. 7. Examine the impact of liberalization on companies owned by Indians. Are they competing with the MNCs satisfactorily? Discuss. Ans: Effects of Globalization on Indian Industry started when the government opened the country's markets to foreign investments in the early 1990s. Globalization of the Indian Industry took place in its various sectors such as steel, pharmaceutical, petroleum, chemical, textile, cement, retail, and BPO. Globalization means the dismantling of trade barriers between nations and the integration of the nations economies through financial flow, trade in goods and services, and corporate investments between nations. Globalization has increased across the world in recent years due to the fast progress that has been made in the field of technology especially in communications and transport. The government of India made changes in its economic policy in 1991 by which it allowed direct foreign investments in the country. As a result of this, globalization of the Indian Industry took place on a major scale. The various beneficial effects of globalization in Indian Industry are that it brought in huge amounts of foreign investments into the industry especially in the BPO, pharmaceutical, petroleum, and manufacturing industries. As huge amounts of foreign direct investments were coming to the Indian Industry, they boosted the Indian economy quite significantly. The benefits of the effects of globalization in the Indian Industry are that many foreign companies set up industries in India, especially in the pharmaceutical, BPO, petroleum, manufacturing, and chemical sectors and this helped to provide employment to many people in the country. This helped reduce the level of unemployment and poverty in the country. Also the benefit of the Effects of Globalization on Indian Industry are that the foreign companies brought in highly advanced technology with them and this helped to make the Indian Industry more technologically advanced. 8. Establish relationship between land reforms, agricultural productivity and elimination of poverty in the Indian economy. Discuss the difficulties in designing and implementation of agriculture friendly land reforms in India. Ans: Analyzing the situation and while establishing a relationship between land reforms, agricultural productivity and elimination of poverty in the Indian economy, it can be seen that how the land holding patterns can affect the productivity. The far-reaching and long-lasting implications of land holding patterns on productivity, growth, and social articulation can be proved by many researches and case studies. As a result, policy makers are increasingly aware that the way in which productive assets and the associated economic opportunities are distributed will have far-reaching implications for long-term development by implementing the land reforms. If high levels of inequality reduce growth, countries that have inherited a

www.careerlauncher.com/news/2013/UPSCmains2013GSanalysis.html

15/27

1/9/14

Civil Services Main Exam 2013


very unequal asset distribution may be able to realize considerable gains from redistribution of assets. This argument, together with evidence of a negative relationship between farm size and productivity, and considerations of social justice has historically formed the justification for a wide range of redistributive land reforms that aimed to create the basis for a more inclusive and sustainable pattern of development. 9. a) Discuss the impact of FDI entry into Multi-trade retail sector on supply chain management in commodity trade pattern of the economy. Ans: Indian retail industry is one of the sunrise sectors with huge growth potential. And, no doubt, introduction of FDI into Multi-trade retail sector has changed the scenario a lot. It has transformed the nature and pattern of supply chain management by allowing the entry of many International players. According to the Investment Commission of India, the retail sector is expected to grow almost three times its current levels to $660 billion by 2015. However, in spite of the recent developments in retailing and its immense contribution to the economy, retailing continues to be the least evolved Industries and the growth of organized retailing in India has been much slower as compared to rest of the world. Undoubtedly, this dismal situation of the retail sector, despite the on going wave of incessant liberalization and globalization, stems from the absence of an FDI encouraging policy in the Indian retail sector. 9. b) Though India allowed Foreign Direct Investment (FDI) in what is called multi-brand retail through the joint venture route in September 2012, the FDI, even after a year, has not picked up. Discuss the reasons. Ans: The finance ministry has proposed that up to 49 per cent foreign direct investment (FDI) in multi-brand retail should be allowed by the automatic route. Though, due to many hindrances, there is slow movement of FDI. For that Govt. has eased the norms. The relaxation of foreign direct investment (FDI) norms in retail certainly augurs well for the growth of the $500 billion organised Indian retail industry. But industry observers don't expect foreign retailers to come and set up shops right away. The relaxation in the FDI norms may create greater interest among foreign investors, but there is always a doubt that it will lead to a surge in international investments. Earlier pre-conditions required foreign retailers to source 30 per cent of their products locally from the time they began operations in India. The new rules allow the figure of 30 per cent local sourcing to be reached over a period of five years. This had become a sore point for foreign retailers. 10. Discuss the rationale behind introducing the Goods and Services Tax (GST) in India. Critically bring out the reasons for the delay in roll out for its regime. Ans: The Kelkar Task Force on implementation of the FRBM Act, 2003 had pointed out that although the indirect tax policy in India has been steadily progressing in the direction of VAT principle since 1986, the existing system of taxation of goods and services still suffers from many problems and had also suggested a comprehensive Goods and Services Tax (GST) based on VAT principle. GST system is targeted to be a simple, transparent and efficient system of indirect taxation as has been adopted by over 130 countries around the world. This involves taxation of goods and services in an integrated manner because the demarcation line between goods and services has blurred, making separate taxation of goods and services untenable. Reasons for delay in GST: The implementation of goods and services tax (GST) is unlikely before 2014-15 as several issues, including tax compensation to states, are yet to be sorted out. Central sales tax, a levy imposed on the inter-state movement of goods, was reduced from 4 per cent to 3 per cent in 2007-08 and to 2 per cent in 2008-09 after the introduction of value-added tax (VAT). The Centre had then promised the states that it would bear the losses on account of the reduction of CST. Earlier this year, the states had agreed that the Centre would bear 100 per cent of the loss incurred by the states in 2010-11. However, for 2011-12 and 2012-13, the Centre will give 75 per cent and 50 per cent of the losses to the states. 11. Write a note on Indias green energy corridor to alleviate the problem of conventional energy. Ans: Germany will provide financial and technical assistance for key segments of the Green Energy Corridors project. German technical assistance includes US$2.7 million for the Indo-German energy program, which is a new component of Green Energy Corridors, and another US$2.7 million for Integration of Renewable Energies into the Indian electricity system. Germany is also willing to grant concessional loans of up to US$1.3 billion for financing the Green Energy Corridors project under the Indo-German Bilateral Development Cooperation Program over the next six years. The Green Energy Corridors project will help in integrating renewable energy into Indias national grid. It consists of both inter-state and intra-state schemes for evacuation of power from wind and solar projects. India said the green energy corridor will facilitate the flow of renewable energy into its grid electricity. The corridor will be built across seven states over the next five to six years. The grid will also receive support from the World Bank and Indias National Electricity Fund. It aims to connect the southern grid to the national grid by 2014 to create the single largest transmission grid in the world. 12. Adoption of PPP model for infrastructure development of the country has not been free of criticism. Critically discuss the pros and cons of the model. Ans: Pitching for Public-Private partnership in infrastructure development, Urban Development Minister Kamal Nath has exemplified Metro operation as the success of the PPP model. The Metro operation is a successful PPP model and it can be adopted in construction of road, port connectivity and other infrastructure projects. Many developing countries like China have developed toll roads and a number of private sector greenfield power projects, while Argentina has developed its power sector mostly through divestiture and greenfield investments. The main aim of the Chinese is to attract investors through PPP. Brazil has not only attracted more greenfield projects in power sector but is also known for its telecommunication sector development under PPP model. The UK provides guarantees for PFI projects to attract more investment. Chile, on the other hand, has succeeded in the PPP model with institutional development, standardised contract procedures and appropriate risk sharing mechanism. Cross country analysis reveals that the PPP model differs widely across countries and sectors. Overall, many developing countries have developed their power projects, roads, telecom, ports and airports through PPP model, which they considered is the apposite way of developing the public infrastructure through private participation, while these countries have faced fiscal constraints. 13. Bringing out the circumstances in 2005 which forced amendment to the section 3(d) in Indian

www.careerlauncher.com/news/2013/UPSCmains2013GSanalysis.html

16/27

1/9/14

Civil Services Main Exam 2013


Patent Law, 1970, discuss how it has been utilized by the Supreme Court in its judgement in rejecting Novartis patent application for Glivec. Discuss briefly the pros and cons of the decision. Ans: There has been considerable press regarding the decision by the Indian Supreme Court in April, 2013, rejecting Novartis patent application for the beta-crystalline form of Imatinib Mesylate (known as Glivec). In fact, many have considered this case to be a test case regarding the judicial interpretation of Section 3(d) of the Indian Patents Act. In view of the significance of this decision, it is imperative for those drafting and prosecuting pharmaceutical and biotech patent applications in India to understand this section of Indian patent law. This is Part I of a two part posting examining Section 3(d). This post examines the genesis of Section 3(d). Part II will provide patent drafting strategies to reduce the risk of receiving a Section 3(d) rejection as well overcoming a rejection received during prosecution and/or an opposition proceeding. The R&D myth The night before the apex court verdict, Novartis threatened to stop investing in research and development in India, if the verdict went against it. How serious is the threat and how realistic is the scenario? In Indias drug production of over Rs. 100,000 crore, Novartis turnover is a little over Rs. 1,000 crore, constituting around one per cent of the total production. Out of the total expenditure of over Rs. 800 crores incurred by Novartis India in 2012, a paltry Rs. 29 lakhs was spent for R&D, constituting roughly 0.03 per cent of its entire expenditure in India. 14. What do you understand by Fixed Dose drug Combinations (FDCs)? Discuss their merits and demerits. Ans: Fixed dose combination (FDC) is a combination of two or more actives in a fixed ratio of doses. This term is used generically to mean a particular combination of actives irrespective of the formulation or brand. It may be administered as single entity products given concurrently or as a finished pharmaceutical product. Recent relevance: The Karnataka High Court has quashed Ministry of Health and Family Welfares notification that suspended manufacture, sale and distribution of a fixed dose combination (FDC) of Flupenthixol and Melitracen, which is a drug used in the treatment of certain neurological and psychiatric disorders. The use of drug was suspended for the reason that it is likely to involve risk to human beings whereas safer alternatives to the said drugs are available based on the May 11, 2013 recommendation made by the New Drug Advisory Committee (NDAC). The NDAC had observed that rationality and essentiality of continued marketing of this FDC is questionable while pointing out that Melitracen is reported to be not efficacious as a single agent in depression, and Flupenthixols use is associated with potentially serious neurological side-effects. 15. What do you understand by Umpire Decision Review System in cricket? Discuss its various components. Explain how silicone tape on the edge of a bat may fool the system? Ans: The Umpire Decision Review System is a technology-based system used in the sport of cricket. The system was first introduced in Test cricket, for the sole purpose of reviewing controversial decisions made by the on-field umpires in the case of a batsman being dismissed or not. The new review system was officially launched by the International Cricket Council on November 24, 2009 during the first Test match between New Zealand and Pakistan at the University Oval in Dunedin. It was first used in One Day Internationals in January 2011, during England's tour of Australia. The ICC had made UDRS mandatory in all international matches but it later decided to end the mandatory use of DRS and now it will be up to both the teams to mutually agree on DRS use. However, the ICC's executive board made it clear that the DRS would still be part of all ICC events and that they support the use of technology and would continue to work on its development. Components: Hawk-Eye, Eagle Eye, or Virtual Eye: ball-tracking technology plots the trajectory of a bowling delivery that has been interrupted by the batsman, often by the pad, and can determine whether it would have hit the wicket or not. Hot Spot: Infra-red imaging system illuminates whether the ball has been in contact with bat or pad. Snickometer relies on directional microphones to detect small sounds made as the ball hits the bat or pad. Role of silicon tape: Recently, Channel Nine News reported that the ICC was looking into claims that batsmen were taping the edges of their blades with silicon tape to fool the Hot Spot cameras. The probe is reportedly focusing on concerns about the caught-behind dismissal of Kevin Pietersen in a match. Snickometer, which is not a part of the third umpire's technological arsenal, showed a clear noise when the ball passed the bat, which was a long way from Pietersen's pad. Hot Spot is part of the DRS that uses infra-red imaging to determine whether the ball made contact with the bat based on heat signatures from friction on impact. 16. a) What is a digital signature? What does its authentication mean? Give various salient built-in features of a digital signature. Ans: Digital signatures are often used to implement electronic signatures, a broader term that refers to any electronic data that carries the intent of a signature, but not all electronic signatures use digital signatures. Digital signatures employ a type of asymmetric cryptography. For messages sent through a nonsecure channel, a properly implemented digital signature gives the receiver reason to believe the message was sent by the claimed sender. There are several reasons to sign such a hash (or message digest) instead of the whole document. For efficiency: The signature will be much shorter and thus save time since hashing is generally much faster than signing in practice. For compatibility: Messages are typically bit strings, but some signature schemes operate on other domains (such as, in the case of RSA, numbers modulo a composite number N). A hash function can be used to convert an arbitrary input into the proper format. For integrity: Without the hash function, the text "to be signed" may have to be split (separated) in blocks small enough for the signature scheme to act on them directly. However, the receiver of the signed blocks is not able to recognize if all the blocks are present and are in the appropriate order. Notions of security 16. b) How does the 3D printing technology work? List out the advantages and the disadvantages of

www.careerlauncher.com/news/2013/UPSCmains2013GSanalysis.html

17/27

1/9/14

Civil Services Main Exam 2013


the technology. Ans: Additive manufacturing or 3D printing is a process of making a three-dimensional solid object of virtually any shape from a digital model. 3D printing is achieved using an additive process, where successive layers of material are laid down in different shapes. 3D printing is also considered distinct from traditional machining techniques, which mostly rely on the removal of material by methods such as cutting or drilling. Uses: The 3D printing technology is used for both prototyping and distributed manufacturing with applications in architecture, construction (AEC), industrial design, automotive, aerospace, military, engineering, civil engineering, dental and medical industries, biotech (human tissue replacement), fashion, footwear, jewelry, eyewear, education, geographic information systems, food, and many other fields. 17. a) What is an FRP composite material? How are they manufactured? Discuss their application in aviation and automobile industries. Ans: Fibre-reinforced plastic (FRP) is a composite material made of a polymer matrix reinforced with fibres. The fibres are usually glass, carbon, basalt or aramid, although other fibres such as paper or wood or asbestos have been sometimes used. The polymer is usually an epoxy, vinylester or polyester thermosetting plastic, and phenol formaldehyde resins are still in use. FRPs are commonly used in the aerospace, automotive, marine, and construction industries. Manufacturing: Carbon fibre production began in the late 1950s and was used, though not widely, in British industry beginning in the early 1960s. Aramid fibres were being produced around this time, appearing first under the trade name Nomex by DuPont. Today each of these fibres is used widely in industry for any applications that require plastics with specific strength or elastic qualities. Glass fibres are the most common across all industries, although carbon-fibre and carbon-fibre-aramid composites are widely found in aerospace, automotive and sporting good applications. 17. b) What do you understand by Run-of-river hydroelectricity project? How is it different from any other hydroelectricity project? Ans: Run-of-the-river hydroelectricity (ROR) is a type of hydroelectric generation whereby little or no water storage is provided. Run-of-the-river power plants may either have no storage at all, or a limited amount of storage, in which case the storage reservoir is referred to as pondage. A plant without pondage has no storage and is, therefore, subject to seasonal river flows and serves as a peaking power plant while a plant with pondage can regulate water flow and serve either as a peaking or base load power plant. Concept: Run-of-the-river hydroelectricity is ideal for streams or rivers with a minimum dry weather flow or those regulated by a much larger dam and reservoir upstream. A dam, smaller than that used for traditional hydro, is required to ensure that there is enough water to enter the penstock pipes that lead to the lowerelevation turbines. Advantages: Traditional hydro dams store enormous quantities of water in reservoirs, necessitating the flooding of large tracts of land. In contrast, most run-of-river projects do not require a large impoundment of water, which is a key reason why such projects are often referred to as environmental friendly, or "green power." 18. How important are vulnerability and risk assessment for pre-disaster management? As an administrator, what are the key areas that you would focus on in a Disaster Management System? Ans: Disaster Planning & Management Suggestive Strategies, Recommendations and Action Plan for Disaster Mitigation, Prevention and Preparedness: Government of India has taken several initiatives for strengthening disaster reduction strategies. Government of India constituted an Expert Group to examine the related issues and evolve recommendations for improving preparedness and prevention with respect to natural disasters caused by earthquakes, floods and cyclones. Key Areas to be focused: The Expert Group appointed by the Govt. of India examined the current status of work being carried out in these areas: Monitoring Hazards Vulnerability Assessment Prediction and Forecasting Retrofitting of Existing Unsafe Structures and Buildings Hazard Mapping Disaster Risk Assessment and Mapping Preparation of Building Guidelines Assessing Gaps in the Above and filling them as much as possible Identification of various hazard prone areas. 19. What are the consequences of Illegal mining? Discuss the Ministry of Environment and Forests

concept of GO AND NO GO zones for coal mining sector. Ans: According to the Lok Ayukta Report there have been severe ecological changes due to illegal mining. Certain species of animals, like the sloth bear that were found in the Bellary region, have disappeared. Medicinal plants of the area do not grow any more. It is reported that the entire area surrounding the mining area is denuded of greenery and has no agricultural activity. For example, illegal mining in Goa is being projected as a bigger scam than Bellary. While revenue losses from illegal mining has been estimated at about Rs 3,000 crore, the loss by way of damage to the environment and loss of livelihood has not been estimated. Many workesrs have lost their source of living. Their farms have been destroyed by mining silt and water sources have been contaminated. Mining has caused irreversible damage to forests, agriculture, fisheries and water aquifers. Goa has 90 operational mines spread along the Western Ghats in an area of 150 to 200 sq. km. Most of the legal and illegal mines are in forest areas. The concept of `go and no go' areas for coal mining is proposed to replace the earlier system of environment and forestry clearance. Under the Forest Conservation Act, 1980, all diversion of forest cover for non-forest uses requires the approval of FAC. The coal ministry earlier considered keeping 10% of thickly forested areas in the "no go" zone and open others for mining after following the due clearance process. 20. Enumerate the National Water Policy of India. Taking river Ganges as an example, discuss the

www.careerlauncher.com/news/2013/UPSCmains2013GSanalysis.html

18/27

1/9/14

Civil Services Main Exam 2013


strategies which may be adopted for river water pollution control and management. What are the legal provisions of management and handling of hazardous wastes in India? Ans: National Water Policy is formulated by the Ministry of Water Resources of the Government of India to govern the planning and development of water resources and their optimum utilization. The first National Water Policy was adopted in September, 1987. It has been reviewed and updated in 2002. India accounts for 15% of the world population and about 4% of the worlds water resources. One of the solutions to solve the countrys water woes is to link the rivers.India has been successful in creating live water storage capacity of about 253 billion cubic meter (BCM). In a first, the ecological needs of river have also been taken into consideration. National Water Policy 2012: Main features The main emphasis of National Water Policy 2012 is to treat water as economic good, which the ministry claims to promote its conservation and efficient use. This provision intended for the privatization of waterdelivery services is being criticized from various quarters. The policy also does away with the priorities for water allocation mentioned in 1987 and 2002 versions of the policy. The policy was adopted with disapproval from many states. 21. Money laundering poses a serious security threat to a countrys economic sovereignty. What is its significance for India and what steps are required to be taken to control this menace? Ans: Emphasizing that India-focused overseas funds are subjected to stringent checks in Mauritius, financial services regulator here said it has stepped up enforcement of norms to prevent any round tripping of funds through the island nation. There are provisions to cancel the license of entities if they are found to be indulging in round-tripping or money laundering activities. To curb money laundering and terror financing activities, the international panel on banking supervision has came out with suggestions, including stronger due diligence and clear policies by banks. The suggestions of Basel Committee on Banking Supervision, whose members include India and the U.S., come amid governments worldwide making substantial efforts to tackle the menace of money laundering and terror funding. Besides strict due diligence, the panel said, banks should have a thorough understanding about inherent money laundering or terror funding risks across customer bases, products and services. Earlier the Reserve Bank of India (RBI) had slapped penalties on three private lenders for violating KYC (know your customer) and anti-money laundering norms and more entities are under the scanner. The consultative document titled Sound management of risks related to money laundering and financing of terrorism has proposed that identity of customers, beneficial owners, as well as persons acting on behalf of customers, should be properly verified. 22. What are social networking sites and what security implications do these sites present? Ans:Social Networking Sites provide a virtual community for people to interact. People need to understand that it will get problematic for them if they reveal too much about their personal lives on a public forum. Four threats Security threats ranges across wider terrain. The international relations theorist Joseph Nye has discerned four different types of threats to cyberspace. The most dramatic is Cyber War the unauthorised invasion by a government into the systems or networks of another, aiming to disrupt those systems, to damage them partially, or to destroy them entirely. A specific target is to slow down if not curtail the military systems of the target state: there is no point having excellent missiles and weapons if the delivery systems can be paralysed. And as our military establishments become more and more dependent on sophisticated technologies, the risk of equally sophisticated attacks on them grows. Cyber Terrorism. This includes websites spreading extremist propaganda, recruiting terrorists, planning attacks, and otherwise promoting terrorists political and social objectives. It also involves the use of hackers by terrorists to debilitate states and governments, much like in Cyber War, with the only difference that this involves a non-State actor. Cyberspace offers a great advantage for the shrouded business of terrorists, making their work murkier than ever to those outside. As weapon Cyber attacks are already happening daily, and as we grow more and more connected, the threats also become more complex. Symantec, a leading international cyber security company, recorded that in 2010 alone there were three billion malware attacks. Of these one stands out especially, pointing to the possible use by legitimate governments of cyber weapons. Social networking websites are also increasingly becoming targets, not only because of the massive databases they provide, but also in order to spread malware that infect computers. On Facebook there are 50 million Indian users and even if a small fraction of them click unsuspectingly on a malevolent but seemingly ordinary link, you have that many computers opened up to risk and infection. Cyber attacks, to state the obvious, can be very personal. Indias response system There are no easy responses to these. The U.S. has created CYBERCOM in 2009 as a military command dedicated to cyber warfare. In the civilian arena few countries have a credible equivalent. Indias own style of dealing with cyber threats leaves much to be desired. It is relatively chaotic and there is a constant insecurity that our cyber-defences are insufficient. This perception has been underscored by frequent reports of successful invasions of Indian cyberspace. 23. Cyber warfare is considered by some defense analysts to be a larger threat than even Al Qaeda or terrorism. What do you understand by Cyber warfare? Outline the cyber threats which India is vulnerable to and bring out the state of the countrys preparedness to deal with the same. Ans: Cyber warfare refers to politically motivated hacking to conduct sabotage and espionage. It is a form of information warfare sometimes seen as analogous to conventional warfare. Espionage and national security breaches Cyber espionage is the act or practice of obtaining secrets (sensitive, proprietary or classified information) from individuals, competitors, rivals, groups, governments and enemies for military, political, or economic advantage using illegal exploitation methods on internet, networks, software and or computers. Classified

www.careerlauncher.com/news/2013/UPSCmains2013GSanalysis.html

19/27

1/9/14

Civil Services Main Exam 2013


information that is not handled securely can be intercepted and even modified, making espionage possible from the other side of the world. Specific attacks on the United States have been given codenames like Titan Rain and Moonlight Maze. General Alexander notes that the recently established Cyber Command is currently trying to determine whether such activities as commercial espionage or theft of intellectual property are criminal activities or actual "breaches of national security. Sabotage Computers and satellites that coordinate other activities are vulnerable components of a system and could lead to the disruption of equipment. Compromising military systems, such as C4ISTAR components that are responsible for orders and communications could lead to their interception or malicious replacement. Power, water, fuel, communications, and transportation infrastructure all may be vulnerable to disruption. According to Clarke, the civilian realm is also at risk, noting that the security breaches have already gone beyond stolen credit card numbers, and that potential targets can also include the electric power grid, trains, or the stock market. Terrorism Eugene Kaspersky, founder of Kaspersky Lab, concludes that "cyber terrorism" is a more accurate term than "cyber war." He states that "with today's attacks, you are clueless about who did it or when they will strike again. It's not cyber-war, but cyber terrorism." He also equates large-scale cyber weapons, such as the Flame Virus and Net Traveler Virus which his company discovered, to biological weapons, claiming that in an interconnected world, they have the potential to be equally destructive. Civil Potential targets in internet sabotage include all aspects of the Internet from the backbones of the web, to the Internet Service Providers, to the varying types of data communication mediums and network equipment. This would include: web servers, enterprise information systems, client server systems, communication links, network equipment, and the desktops and laptops in businesses and homes. Electrical grids and telecommunication systems are also deemed vulnerable, especially due to current trends in automation. Private sector Computer hacking represents a modern threat in ongoing industrial espionage and as such is presumed to occur widely. It is typical that this type of crime is underreported. According to McAfee's George Kurtz, corporations around the world face millions of cyber attacks a day. "Most of these attacks dont gain any media attention or lead to strong political statements by victims." This type of crime is usually financially motivated. 24. Article 244 of the Indian Constitution relates to administration of schedules area and tribal areas. Analyse the impact of non-implementation of the provisions of the Fifth schedule on the growth of Left Wing extremism. Ans: Part IX concerning Panchayats was added to the Constitution in 1993 by the 73rd Amendment Act. The 73rd amendment was the first ever law after independence which did not cover Scheduled Areas in routine as it was. Accordingly, a high powered Committee comprising select Members of Parliament and Experts was appointed in 1994 to recommend exceptions and modifications that may be made in Part IX in its application to the SA. The Committee submitted its Report in 1995. The Government of India generally accepted its recommendations. The Provisions of Panchayats (Extension to the Scheduled Areas) Act (PESA) was enacted in 1996. The provisions of Part IX of the Constitution were extended to SA subject to the special features mentioned in Section 4. Since British times, tribals have been denied certain rights which they had enjoyed since antiquity. Post independence this policy continued and resulted in open conflict between the government and the tribals. The crucial aspects of governance relating to management of natural resources, the general economic system or even quasi-social matters like use of intoxicants, had been appropriated by the formal legal system. In this confused situation the message of PESA aroused great expectations amongst the tribal people across the country. 25. How far are Indias internal security challenges linked with border management particularly in view of the long porous borders with most countries of South Asia and Myanmar? Ans: India has landmass almost equal to the sub-continental proportions and occupies strategic position in South Asia. Indias land border with Bangladesh is around 4339 km, Bhutan 605 Km, China 3439 Km, Myanmar 1425 Km, Nepal 1690 Km and Pakistan 3325Km (Afghanistan 106 Km, now part of Northern Areas of POK). In addition it also has a coastline running through 13 states and union territories, of approximately 7600 Km long. The territorial and boundary disputes with Pakistan and China, combined with porous eastern borders have made effective border management a great challenge for Indian security forces. There is no doubt that the development and nation building challenges and internal security are linked with border security and border management. The external threats to Indias internal security are not the only border management issues dealt with by the national security apparatus, but Indias growth rate has outpaced most of its neighbors and this has generated problems like mass migration in India. Other threats have also emerged. The major security concerns of India in this context are:Trans-border terrorism and movement of insurgents particularly in the light of the terrorist attacks on the major cities of India. Illegal migration has changed the demography of the north-east and is one of the main causes for the rise of rebel groups in that area. Emergence of non-state actors like terrorist organizations, religious groups and illegal immigrant groups. Nexus between arms and narcotics smugglers as Northeast India has close proximity to the Golden Triangle. Most weapons of the rebel groups of north-east India, including AK-47 and AK-56 assault rifles, mortars, 40 mm rocket launchers, pistols, revolvers and grenades are bought from Coxs Bazar in Bangladesh and come through Bhutan, Myanmar, Thailand and Cambodia via the land and sea route. Separatist movement aided and abetted by Pakistan. Establishment of madrasas particularly in the border areas, that has become the recruiting place for the terrorist outfits.

www.careerlauncher.com/news/2013/UPSCmains2013GSanalysis.html

20/27

1/9/14

Civil Services Main Exam 2013


China is supplying arms and ammunition to northeast insurgent groups through Myanmar. Some of these weapons are further finding their way to the Naxalites. Earlier LTTE had supplied weapons to the Indian Left Wing Extremists (LWE) through the Indian coastal borders. It is also a fact that some of them had even joined hands with LWE and are provided expertise training to the Naxalites.

GENERAL STUDIES (Paper IV)


Section- A What do you understand by values and ethics? In what way is it important to be ethical along with being professionally competent? Ans:Ethics is concerned with criteria of what is morally right and wrong. It includes the formulation of moral rules that have direct implications for what human actions, institutions, and way of life should be like. Values are the accepted principles or standards of an individual or a group. Values are like an anchor in a ship. When a storm comes, a ship is not swept away by strong currents but remains anchored to the shore. A person with strong values or character sticks to his principles and is not swept away, in crisis or under pressure. Ethically justified action requires that the individual has the ability to consider different alternatives and to place himself in the position of the other person (empathy). Good discussions on ethics are often driven by situations that challenge ones abilities to determine the right thing to do, carry out effective ethical action, or lay out an effective strategy for avoiding ethical obstacles in the future.

Relationship between ethical conduct and professional competence It is very important to be ethical and professionally competent. Infact, it is more important to be ethical than professional as basic values should never be compromised at any cost. Following factors justify the magnitude of the ethics in a contemporary public administration, more concretely: material and resource difficulties needs of a growing and multifaceted society critical and attentive perspective implied in citizenship restructuring subjects and change of the administration constant demand that decision processes become more thoroughly participated Being ethical is much broader than being merely a financial propriety. Not accepting any monetary benefits for any work is certainly a basic segment of ethics, yet there are many more aspects of this phenomenon. Observance of compassionate attitude, intellectual integrity, keeping interest of society above the self, taking decisions keeping in mind the supremacy of the constitution and the interest of vulnerable sections of society, respecting dignity of the common citizen, being responsive, remaining transparent are all equally important components of an ethical order. 2. a) What do you understand by the following terms in the context of public service? Integrity Integrity means holders of public office should not place themselves under any financial or other obligation to outside individuals or organizations that might influence them in the performance of their official duties. Integrity means recognition of the need to be true to one's own thinking; to be consistent in the intellectual standards one applies; to hold one's self to the same rigorous standards of evidence and proof to which one holds one's antagonists; to practice what one advocates for others; and to honestly admit discrepancies and inconsistencies in one's own thought and action. So integrity in public services means following the highest standards of evidence and proof while formulating or implementing the decision by an administrator and honestly admitting any discrepancies or inconsistencies even if done with bonafide intentions. Perseverance Perseverance means continuing in a course of action without regard to discouragement, opposition or previous failure. In other words, perseverance can be defined as persistent determination to adhere to a plan of direction. So, perseverance in public life means adhering to a certain course of action without discouragement when some project is not showing off the desired results even with the best intentions. Various government projects are not showing off the desired results for one reason or the other. So perseverance helps the public servants to identify the challenges and address them. Despite facing sociopolitical or economic challenges, a public servant continues to work. Spirit of service Spirit of service envisages that holders of public office should take decisions solely in terms of public interest. They should not do so in order to gain financial or other material benefits for themselves, their family or their friends. There is general feeling among people of a lack of the spirit of service expected of the members of the public services and also of the dilatory methods and tactics in their dealings with the public. These lapses on the part of the public services very often compel the public to seek the intervention of legislators or public men of importance for the disposal of even matters of routine nature. Thus, the first obligation of public servants is to render service to, and not merely to exercise authority over, the public. An improvement in the attitude and conduct of services towards the common man is necessary for peoples active cooperation in the stupendous task of building the nation through developmental planning and its implementation; and this improvement in their attitude and conduct should be visible to the common man. Commitment

www.careerlauncher.com/news/2013/UPSCmains2013GSanalysis.html

21/27

1/9/14

Civil Services Main Exam 2013


Commitment is the individual's psychological attachment to the organisation and its work. The basis behind many of the studies was to find ways to improve how workers feel about their jobs so that these workers would become more committed to their organizations. Some of the factors such as role stress, empowerment, job security and employability, and distribution of leadership have been shown to be connected to a worker's sense of organizational commitment. Organizational commitment can be contrasted with other work-related attitudes, such as job satisfaction (defined as an employee's feelings about their job) and organisational identification (defined as the degree to which an employee experiences a 'sense of oneness' with their organization). Courage of conviction Courage of conviction refers to the quality of mind or spirit that enables a person to face difficulty, danger, pain, etc., without fear. In administration, this attribute plays a decisive role. Administrators working in todays working environment are confronted with increasingly complex moral and ethical dilemmas. Administrators encounter these dilemmas in situations where their ability to do the right thing is frequently hindered by conflicting values and beliefs of other colleagues. In these circumstances, upholding their commitment to larger public interest requires significant moral courage. Administrators who possess moral courage and advocate in the best interest of the public may at times find themselves experiencing adverse outcomes. These issues underscore the need for all administrators in all roles across all settings to commit to working toward creating work environments that support moral courage. b) Indicate two more** attributes which you consider important for public service. Justify your answer. Though there are several attributes which are very important for public servants, two important attributes are devotion to work and accountability. a) Devotion to Work: Devotion to work means ardent, often selfless affection and strong attachment to or affection towards work. A devoted employee is self-motivated, requires fewer instructions and has complete trust on the authorities. Working for long hours does not necessarily mean dedication. A devoted employee would rather put his/her heart and soul in his/her work. A devoted employee shows effective commitment towards his/her work. As stated by Dr. A.P.J. Kalam, To succeed in your mission, you must be single-minded devoted to your goal. b) Accountability is essential to the field of public administration, and particularly important within the context of administrative discretion. Effective administration requires administrators to exercise discretion, and the exercise of discretion requires accountability. The question then becomes, to whom and for what are public administrators accountable. 3. Some people feel that values keep changing with time and situation, while others strongly believe that there are certain universal and eternal human values. Give your opinion in this regard with proper justification. Ans: Values are usually viewed as relatively stable. Personal values are the strongest beliefs that a person adheres to. They serve as a guiding basis for every decision in a persons life. Since personal values are so important in shaping the individual, should they change over time? The answer to this question is both yes and no. Life experience, understanding about the world around a person, and cultural influences all shape a persons values and identity. In certain cases it is natural to change values over time. For example, the cares of ones youth and how they see the world often changes quite dramatically once they have a family or garner a better understanding of the world around them. Religion is another area that changes a person, sometimes drastically. While some values naturally change core values should not. Having a well- established understanding of right and wrong helps to ground an individual and prevent moral lapses. For example, murder, stealing, and rape being wrong are fundamental core values that should not change over time. There are many other such values like the need for laws and governments, respect for elders and a good work ethic. 4. What is emotional intelligence and how can it be developed in people? How does it help an individual in taking ethical decisions? Ans: Emotional Intelligence Quotient is defined as a set of competencies demonstrating the ability one has to recognize such as his or her behavior, moods, and impulses, and to manage them best according to the situation. Typically, "emotional intelligence" is considered to involve emotional empathy; attention to, and discrimination of one's emotions; accurate recognition of one's own and others' moods; mood management or control over emotions; response with appropriate (adaptive) emotions and behaviors in various life situations (especially to stress and difficult situations); and balancing of honest expression of emotions against courtesy, consideration, and respect (i.e., possession of good social skills and communication skills). Use of EI in HR Applications Human resources (HR) departments use more emotional competencies based applications. There are basically two ways to increase emotional intelligence in an organization: Hire people who are emotionally intelligent. Develop emotional intelligence of the current employees. II. Emotional Intelligence Is Linked to Performance. Its a powerful way to focus your energy in one direction with a tremendous result. It has been found that emotional intelligence is the strongest predictor of performance, explaining a full 58% of success in all types of jobs. a) What do you understand by the term 'voice of conscience'? How do you prepare yourself to heed to the voice of conscience? Conscience is an aptitude, faculty, intuition or judgment that assists in distinguishing right from wrong. Moral judgment may derive from values or norms (principles and rules). Conscience is the voice of the Self which says yes or no when you are involved in a moral struggle. It raises the voice of protest whenever

www.careerlauncher.com/news/2013/UPSCmains2013GSanalysis.html

22/27

1/9/14

Civil Services Main Exam 2013


anything is thought of or done contrary to the interest of its master. Conscience is a form of truth. It is a sensitive balance to weigh actions. It is the faculty or principle by which we distinguish right from wrong. How to prepare one to heed to the voice of conscience To hear voice of conscience, one has to practice meditation and concentration. Practice means repetition of the same thing again and again to form a strong habit. Mind is different from conscience. Conscience always tries to persuade to improve self and unfold inner being, whereas mind always tries to persuade not to practice. Conscience is like a mirror that shows you your face exactly as it is. What is meant by 'crisis of conscience'? Narrate one incident in your life when you were faced with such a crisis and how you resolved the same. (150 words) Conscience is obscured through human weakness. The rumbling of emotional conflicts, base impulses and dictates of the flesh stifle the faint inner voice of the spirit. It is awakened and sharpened to new clearness through purity of conduct and practice of ethical virtues. One must carefully watch their thoughts, speech and actions. One becomes insincere on account of intoxication from selfishness and greed due to clouded understanding. Most people are ignorant about such thought processes. If one takes bribe, their thoughts and actions are registered in the subconscious mind and leads to loss of ethics and virtues in longer run of life. A remedy for this can be reducing ones wants and living honestly within means. This will lead to clean conscience and freedom from anxieties and worries. 6. Given below are three quotations of great moral thinkers/philosophers. For each of these quotations, bring out what it means to you in the present context: "There is enough on this earth for every one's need but for no one's greed." Mahatma Gandhi. India's great leader Mohandas Gandhi famously said that there is enough on Earth for everybody's need, but not enough for everybody's greed. Today, Gandhi's insight is being put to the test as never before. The world is hitting global limits in its use of resources. We are feeling the shocks each day in catastrophic floods, droughts, and storms - and in the resulting surge in prices in the marketplace. Our fate now depends on whether we cooperate or fall victim to self-defeating greed. The limits to the global economy are new, resulting from the unprecedented size of the world's population and the unprecedented spread of economic growth to nearly the entire world. There are now seven billion people on the planet as compared to just three billion a half-century ago. Today, average per capita income is $10,000, with the rich world averaging around $40,000 and the developing world around $4,000. Yet there is another side to the global growth story that we must understand. Our planet will not physically support this exponential economic growth if we let greed take the upper hand. Even today, the weight of the world economy is already crushing nature, rapidly depleting the supplies of fossil-fuel energy resources that nature created over millions of years ago, while the resulting climate change has led to instabilities in terms of rainfall, temperature, and extreme storms. "Nearly all men can withstand adversity, but if you want to test a man's character, give him power." Abraham Lincoln Ans: Abraham Lincoln said, " Nearly all men can withstand adversity, but if you want to test a man's character, give him power". The misuse of power phenomena has been there through out the history of mankind. It has led to corruption, coercion and all the misdeeds in the society. Many experts have given the solution but how to implement it in our society is the question. One probable method could be by introducing the subject in school curriculum and by extensive media participation. Another view is that it is not that power corrupts a man, but man is himself corrupt. Power only brings his corruption into the open. Suppose someone wanted to kill somebody, but he did not have the power to kill; and in the next instance he gets the power, he will kill. It is not power that corrupts a person; power simply gives the opportunity to do whatever he wants to do. "I count him braver who overcomes his desires than him who overcomes his enemies."Aristotle Ans: To me, this is talking about the eternal internal struggle of humanity. Overcoming ones desires is a very difficult thing for most of us to do. Conquering an enemy or defeating any external obstacle is easy when compared to conquering that which lies within us. With everything focused on the outside, one can ignore the internal conflict. When facing self, one must bring a level of honesty and integrity that is very admirable but more than a little difficult. Self-discipline begins with self-knowledge. Self-knowledge is necessary to understand motivations and desires. So to overcome desires, one must start with self-knowledge and then can move to self-discipline. Later, with practice, comes the victory over ones desires. Lastly, Self-knowledge and self-exploration is a journey, not a destination. 7. "The good of an individual is contained in the good of all." What do you understand by this statement? How can this principle be implemented in public life? Ans: Mahatma Gandhi had made his first major statement on socio-economic order in his paraphrase of John Ruskins Unto This Last (1860) in his booklet titled Sarvodaya (Welfare of all, 1908). He summed up his understanding of the lessons of Ruskins book in his Autobiography as: That the good of the individual is contained in the good of all. To Gandhi the spirit of service and sacrifice was the key to leadership. For the spirit of service to materialize, we must lay stress on our responsibilities and duties and not on rights. He illustrated it through the example of concentric circles: one starts with service of those nearest to one and expands the circle of service until it covers the universe, no circle thriving at the cost of the circles beyond. Service to him implied self-sacrifice. 8. It is often said that 'politics' and 'ethics' do not go together. What is your opinion in this regard? Justify your answer with illustrations. Political attitudes are the attitudes of people to the areas of public life covered by political psychology. For example, views on nationalism, political conservatism, political liberalism, political radicalism etc. If morality is

www.careerlauncher.com/news/2013/UPSCmains2013GSanalysis.html

23/27

1/9/14

Civil Services Main Exam 2013


the answer to the question 'how ought we to live' at the individual level, politics can be seen as addressing the same question at the social level. It is therefore unsurprising that evidence has been found of a relationship between attitudes in morality and politics. Many political parties base their political action and program on an ideology. In social studies, a political ideology is a certain ethical set of ideals, principles, doctrines, myths or symbols of a social movement, institution, class, or large group that explains how society should work, and offers some political and cultural blueprint for a certain social order. A political ideology largely concerns itself with how to allocate power and to what ends it should be used. Some parties follow a certain ideology very closely, while others may take broad inspiration from a group of related ideologies without specifically embracing any one of them. Section B In the following questions, carefully study the cases presented and then answer the questions that follow: 9. A Public Information Officer has received an application under RTI Act. Having gathered the information, the PIO discovers that the information pertains to some of the decisions taken by him, which were not found to be right altogether. There were also other employees, who were party to these decisions. Disclosure of the information is likely to lead to disciplinary action with possibility of punishment against him as well as some of his colleagues. Non-disclosure or part disclosure or camouflaged disclosure of information will result into lesser punishment or no punishment. The PIO is otherwise an honest and conscientious person but this particular decision, on which the RTI application has been filed, turned out to be wrong. He comes to you for advice. The following are some suggested options. Please evaluate the merits and demerits of each of the options: The PIO could refer the matter to his superior officer and seek his advice and act strictly in accordance with the advice, even though he is not completely in agreement with the advice of the superior. Ans: Merit of this decision is that PIO can rely on his senior for his support if anything goes wrong as PIO has taken a decision suggested by his senior. But it may not be ethical to take a decision which one is not convinced of. Moreover, since the decision is going to affect the PIO directly, PIO should be convinced before taking any decision and should take an independent stand considering all the opinions. 2. The PIO could proceed on leave and leave the matter to be dealt by his successor in office or request for transfer of the application to another PIO. Ans: The PIO could proceed on leave and leave the matter to be dealt by his successor in office or request for transfer of the application to another PIO is a decision which may possibly relieve the PIO for the time being. But his successor may take a decision to make the documents public. Moreover, it is ethically wrong to avoid uncomfortable yet important situations. 3. The PIO could weigh the consequences of disclosing the information truthfully, including the effect on his career, and reply in a manner that would not place him or his career in jeopardy, but at the same time a little compromise can be made on the contents of the information. Ans: This option is like playing safe. Again it may relieve a PIO from any adverse action. But it is against the work ethics to compromise on the contents of the information. Moreover, deliberately hiding information is punishable under RTI act. Also, full information can be disclosed later on which can seriously damage the honest and conscientious image of the PIO. 4. The PIO could consult his other colleagues who are party to the decision and take action as per their advice. Ans: Consulting the colleagues is fine. Advice tendered may or may not be ethical. PIO should disclose the information completely without compromising on any information or content. The PIO is otherwise an honest and conscientious person but this particular decision, on which the RTI application has been filed, turned out to be wrong. This shows that he had taken a decision in a bonafide manner and not in a malafide manner. He should convey the same. Moreover, to err is human. A person should be punished for intentional wrong doings.

10. You are working as an Executive Engineer in the construction cell of a Municipal Corporation and are presently in-charge of the construction of a flyover. There are two Junior Engineers under you who have the responsibility of day-to-day inspection of the site and are reporting to you, while you are finally reporting to the Chief Engineer who heads the cell. The Junior Engineers have been regularly reporting that all construction is taking place as per design specifications. While the construction is heading towards completion, in one of your surprise inspections, you have noticed some serious deviations and lacunae, which, in your opinion, are likely to affect the safety of the flyover. Rectification of these lacunae at this stage would require a substantial amount of demolition and rework which will cause a tangible loss to the contractor and will also delay completion. There is a lot of public pressure on the corporation to get this construction completed because of heavy traffic congestion in the area. When you brought this matter to the notice of the Chief Engineer, he advised you that in his opinion it is not a very serious lapse and may be ignored. He advised for further expediting the project for completion in time. However, you are convinced that this was a serious matter which might affect public safety and should not be left unaddressed. What will you do in such a situation? Some of the options are given below. Evaluate the merits and demerits of each of these options and finally, giving reasons, suggest what course of action you would like to take. Follow the advice of the Chief Engineer and go ahead. Ans: Following the advice of the Chief Engineer and going ahead means that I am following the orders as per organizational hierarchy. This may save me from adverse action in the future if anything unfortunate happens. Moreover, the project should also be completed in time. But since I am convinced that this is a serious matter, which might affect public safety and should not be left unaddressed, ethically I should not let the issue go unaddressed. This will be serious breach of my official

www.careerlauncher.com/news/2013/UPSCmains2013GSanalysis.html

24/27

1/9/14

Civil Services Main Exam 2013


duty. Moreover, accepting oral advice of chief engineer can put me in a problem in the future, as onus of responsibility will come on me. Make an exhaustive report of the situation bringing out all facts and analyses along with your own viewpoints stated clearly and seek for written orders from the chief Engineer. Ans: Making an exhaustive report of the situation bringing out all facts and analyses along with your own viewpoints stated clearly and seeking for written orders from the chief Engineer means that I will be putting all my concerns on writing and asking for the seniors orders on the given issue. Here I will be fulfilling my duty to address the issue to the senior. But here I am not taking an independent stand. I am fulfilling the duty of an employee but not of a public servant because I am convinced about the seriousness of the matter. Call for explanation from the Junior Engineers and issue orders to the contractor for necessary correction within targeted time. Ans: Calling for explanation from the Junior Engineers and issuing orders to the contractor for necessary correction within targeted time means that I will be taking an independent stand on the issue. Moreover, Junior Engineers have been regularly reporting that all construction is taking place as per design specification, so asking for explanation from them is a necessary. At the same time, remedial measures like asking the contractor for necessary correction within targeted time can be helpful. But this action may delay the completion of the work. Highlight the issue so that it reaches superiors above the Chief Engineer. Ans: Highlighting the issue so that it reaches superiors above the Chief Engineer may bring the issue into the notice of higher authorities. Moreover, I will not be held liable for any delay or untoward incident happening in the future. But it may create speculation and can cause un-necessary harm to the institution. Moreover, the most important work is to ensure public safety, so immediate necessary steps need to be taken. So, remedial measures need to be taken without waiting for anyones order so that any further loss can be avoided. Considering the rigid attitude of the Chief Engineer, seek transfer from the project or report sick. Ans: Considering the rigid attitude of the Chief Engineer, seeking transfer from the project or reporting sick can save me from any adverse action if any unfortunate incident happens in the future. But it is totally unethical on my part to risk the public safety at any cost, be it my job. Moreover as a public servant, I should handle these situations in an intelligent manner so that public safety, which is of prime importance, shall never be compromised 11. Sivakasi in Tamil Nadu is known for its manufacturing clusters on firecrackers and matches. The local economy of the area is largely dependent on the firecracker industry. It has led to tangible economic development and improved standard of living in the area. So far as child labour norms for hazardous industries like firecracker industry are concerned, International Labour Organization (ILO) has set the minimum age for admission to employment as 18 years. In India, however, this age is 14 years. The units in industrial clusters of firecrackers can be classified into registered and non-registered entities. One typical unit is household-based work. Though the law is clear on the use of child labour employment norms in registered/non-registered units, it does not include household-based works. Household-based work means children working under the supervision of their parents/relatives. To evade child labour norms, several units project themselves as household-based works but employ children from outside. Needless to say that employing children saves the costs for these units leading to higher profits to the owners. On your visit to one of the units at Sivakasi, the owner takes you around the unit which has about 10-15 children below 14 years of age. The owner tells you that in his household-based unit, the children are all his relatives. You notice that several children smirk, when the owner tells you this. On deeper enquiry, you figure out that neither the owner nor the children are able to satisfactorily establish their relationship with each other. a) Bring out and discuss the ethical issues involved in the above case. Ans: Child labour is a very important problem in India and the world. The term child labour is often defined as work that deprives children of their childhood, their potential and their dignity and that is harmful to their physical and mental development. As minors they are considered incapable of running their own affairs, which are left to adults, normally parents, to act as guardians for them. When the legal system intersects with a childs life, it normally does so in a distinct way from adults. This includes juvenile delinquency, due process for children involved in the criminal justice system, appropriate representation and effective rehabilitative services. Moreover, as per the recent regulations of the government of India, employing children below 14 years of age is completely banned in all occupations and processes. Employment of children as domestic help and at road-side eateries, restaurants and tea stalls was banned in 2006. So, there is no question of employing children below 14 years of age even in household-based works. b) What would be your reaction after your above visit? Ans: Response:a. Licenses of all units of manufacturing clusters of firecrackers and matches should be reviewed thoroughly. b. Look for details of all employees of these units like birth certificate, aadhar card or any other age proof. These should be submitted to the concerned authorities and should be thoroughly re-checked. c. If there is absence of birth certificates or any other age-proof certificates, then age should be confirmed through appropriate medical examination. d. Those employees who are found to be less than 14 years of age should be either sent to their parents or orphanages. All of them should be provided with education and health facilities. Since most of them

www.careerlauncher.com/news/2013/UPSCmains2013GSanalysis.html

25/27

1/9/14

Civil Services Main Exam 2013


belong to poor families, their employability is also important for their survival in the future. So, vocational training courses should be started so that adequate skills can be imparted to them. e. Those employers who are found guilty of employing children of less than 14 years of age should be prosecuted as per law. Moreover, their licenses should be cancelled. Strict warning should be issued regarding the same. f. Proper mechanism, using e-governance, should be put in place to regulate the concerned sector. g. Orders should be issued to concerned authorities to go for regular inspections. 12. You are heading a leading technical institute of the country. The institute is planning to convene an interview panel shortly under your chairmanship for selection for the post of professors. A few days before the interview, you get a call from the Personal Secretary (PS) of a senior government functionary seeking your intervention in favour of the selection of a close relative of the functionary for this post. The PS also informs you that he is aware of the long pending and urgent proposals of your institute for grant of funds for modernization, which are awaiting the functionarys approval. He assures you that he would get these proposals cleared. a) What are the options available to you? Ans: Since this case study is particularly related with nepotism in job recruitment, and this practice is unhealthy as well as unethical for any institution or organization, I have following options in order of precedence: (i) With immediate effect, I will bring lured offer of PS to my senior officials and demand them to empower me to take any rational decision as per my discretion. (ii) Seeking majority in the panel, I will make all the members of the panel aware about this issue. (iii) I will refer the incident of baiting to ethical committee of the department. (iv) As far as the pending funds are concerned, I will look into the matter separately and will write to the concerned department. (v) Under extreme pressure, I will stand against the proposal and without any hesitation, I will resign to oppose the offer. b) Evaluate each of these options and choose the option which you would adopt, giving reasons. Ans: Reasons for each and every explanation: (i) Nepotism is breeding corruption in India and it is destroying the intelligent network. Hence, there is an urgent need to review the stand and take a more holistic and compassionate approach and implore management to retract the dismissal of such practice. In this case, with immediate effect I will bring the matter to the knowledge of my senior officials and would ask them to empower me so that I can turn down and over ride such bait. (ii) Since I am the chairman of the panel, any decision made by me should not send any message of hypocrisy. In such a case, other members of the panel must come to know about such ill offers so that I can seek a favor and support from them. (iii) Following the aforesaid options, my next step will be to bring the event under the light of ethical committee of the department so that I can curb any such practices and hence, discourage all such similar incidents from happening. (iv) As far as the pending funds are considered, they have nothing to do with any immoral negations. Moreover, there is no guarantee that if I agree with the PS, the funds will meet the demands of the department. (v) At last, if I am not able to seek the appropriate co-operation and favor from my department, seniors or sub-ordinates, I will resign rather than kneeling down and favoring the mal practice. 13. As a senior officer in the Finance Ministry, you have access to some confidential and crucial information about policy decisions that the Government is about to announce. These decisions are likely to have farreaching impact on the housing and construction industry. If the builders have access to this information beforehand, they can make huge profits. One of the builders has done a lot of quality work for the Government and is known to be close to your immediate superior, who asks you to disclose this information to the said builder. a) What are the options available to you? Ans: Probity in governance is an essential and vital requirement for an efficient and effective system of governance and for socioeconomic development. An important requisite for ensuring probity in governance is absence of corruption. Following options can be considered: (i) Appointment of a team of experts to evaluate all the tenders objectively. (ii) Informing the vigilance department about the builder. (iii) Speaking out against the unethical practices b) Evaluate each of these options and choose the option which you would adopt, giving reasons. Ans: (i) Appointment of a team of experts to evaluate all the tenders objectively. Clear instructions should be given to the local officials that objective analysis of tender should be done. In case of any discrepancy, respective officials should be held liable. Reports should also be put in the public domain for public scrutiny (ii) Informing the vigilance department about the builder. Under a Westminster-type system of government an employees views are not supposed to take precedence over government policy. Yet, if the employee considers an instruction to be unreasonable or unlawful, there is an expectation that he or she should refuse to comply with it. This interpretation picks up on the notion of public servants being motivated by a duty to serve the wider public interest. In my view, the issue should be brought into the notice of the vigilance department/auditors of the government because public officials are expected to act in `the public interest. It is similarly expected that those public officials who control the financial and other resources provided by the community have an ethical obligation to ensure that those resources are used efficiently and appropriately. The use and misuse of the resources raises an important question of professional ethics. (iii) Speaking out against the unethical practice I would speak out against this unethical practice, if superiors do not promote objectivity. Not only is the organisational culture not conducive to ethical behaviour and practices, but also supervisors lacked the type

www.careerlauncher.com/news/2013/UPSCmains2013GSanalysis.html

26/27

1/9/14

Civil Services Main Exam 2013


of leadership that is essential for promoting, supporting and building an ethical public service. Unless widespread problems like corruption are tackled, there is every possibility that the same problems that plagued the delivery of services by the public sector could impair the functioning of public-private partnerships. 14. You are the Executive Director of an upcoming InfoTech Company which is making a name for itself in the market. Mr. A, who is a star performer, is heading the marketing team. In a short period of one year, he has helped in doubling the revenues as well as creating high brand equity for the Company so much so that you are thinking of promoting him. However, you have been receiving information from many corners about his attitude towards the female colleagues, particularly his habit of making loose comments on women. In addition, he regularly sends indecent SMSs to all the team members including his female colleagues. One day, late in the evening, Mrs. X, who is one of Mr. As team members, comes to you visibly disturbed. She complains against the continued misconduct of Mr. A, who has been making undesirable advances towards her and has even tried to touch her inappropriately in his cabin. She tenders her resignation and leaves your office. a) What are the options available to you? Ans a) Being very effective and hard working at work place does not make a person free from any kind of criminal liabilty. The issue comes under the act of sexual harassment at work place. A committee with appropriate women members should be constituted to investigate the matter within a specified time frame and should be told to submit a report therein. b) Evaluate each of these options and choose the option you would adopt, giving reasons. Ans: Sexual harassment results in violation of the fundamental rights of a woman to equality under articles 14 and 15 of the Constitution of India and her right to life and to live with dignity under article 21 of the Constitution and right to practice any profession or to carry on any occupation, trade or business under article 19(1)(f), which includes a right to a safe environment free from sexual harassment. If Mr. A is found guilty, a complaint should be immediately lodged with the local police under the Sexual Harassment of Women at Workplace (Prevention, Prohibition and Redressal) Act, 2013 that seeks to protect women from sexual harassment at their place of work.

Test Prep Programs


For School Students
BBA CA-CPT Engineering English Language Training Future Map Hotel Management Medical Law

For College Students


Banking Campus Express Civil Services GATE MBA

International Education
GMAT GRE SAT

K-12 Education
Indus World Schools

Other Verticals

Higher Education
Indus World School of Business

Vocational Education
Skill School Kestone

Home

About CL Educate

E Center

Contact Us

Blog

CL Franchise Option

Sitemap

www.careerlauncher.com/news/2013/UPSCmains2013GSanalysis.html

27/27

You might also like